Categories
English English language Etymology Expression Language Pronunciation Usage Word origin Writing

A puny subject

Q: I love Victorian novels and often read them on my Kindle. I just came across “puisne” in Vanity Fair. When I highlight the word, the dictionary tells me it’s a junior judge and pronounced like “puny.” Are these two terms related?

A: Not only are “puny” and “puisne” related, but they were essentially the same word when borrowed from puisné, Middle French for “younger.”

The Middle French term, a compound of puis (later) and né (born), is spelled puîné in modern French. The Chambers Dictionary of Etymology says the usage ultimately comes from the classical Latin post (after) and nātus (born).

When the word entered English in the 1500s as a noun and an adjective, it was spelled various ways: punie, punee, puine, puisne, and so on—all pronounced “puny,” an anglicized version of the French pronunciation.

The Oxford English Dictionary says the English term was originally a noun for a new or junior student. The earliest OED citation is from a 1548 book by the historian William Patten about a trip that Prince Edward, Duke of Somerset, made to Scotland:

“Like yplay in Robin Cooks skole, whear bicaus the punies may lerne thei strike fewe strokes, but by assent & appointement.” (The new students were learning their assigned strokes at Robin Cook’s fencing school.)

When the word first showed up in print as an adjective, it meant “younger” or “junior.” The earliest OED example, which we’ve expanded, is from The Apologie of Fridericus Staphylus, Thomas Stapleton’s 1565 translation of a 1558 Latin treatise by the German theologian Friedrich Staphylus:

“Neither may you M. Grindall be offended herewith, when you shall vnderstand it, as I wish you maye, iff a young scholer and puine student in diuinite aduenter to encounter with you.” (Staphylus, a Protestant convert to Roman Catholicism, is addressing Edmund Grindal, an English clergyman who would later become Archbishop of Canterbury.)

Later in the 1500s, the noun came to mean an inexperienced person, an inferior, a subordinate, someone of no importance, and a junior judge.

All those senses are now obsolete or rare except for the use of the term (now spelled “puisne”) in reference to a subordinate judge or justice, according to the OED.

The dictionary’s earliest example for the adjective used in this sense is from The Common Welth of England (1589), by Sir Thomas Smith:

“The officer before whom the Clarke is to take these essoines, is the puny Iustice in the common pleas.” (The “essoines” here are excuses for not appearing in court on time before the subordinate justice.)

As for the noun, the first Oxford citation for this sense is from Skialetheia, or The Shadowe of Truth, a 1598 collection of poetry by Edward Guilpin:

“Oh he’s a puisne [lesser judge] of the Innes of Court, / Come from th’ Vniuersity to make sport.”

Since the mid-1600s, the term (now always spelled “puisne” and pronounced “puny”) has been used in the UK and some former British dependencies to designate “any judge, justice, etc., other than the most senior in the higher courts of law,” according to the dictionary.

The first OED citation refers to a case argued before “Mallet the puisne Judge” (from a 1648 collection of legal cases, compiled by the English barrister John March).

In Vanity Fair, the Thackeray novel that prompted your question, the term is similarly used to describe a less than senior judge. The novel, which was serialized in Punch from 1847 to ’48, describes Lady Smith as the “wife of Sir Minos Smith the puisne judge.”

Finally, we have Shakespeare to thank for the use of “puny” as an adjective meaning small, weak, or insignificant, the usual sense of the word today.

The Chambers etymological dictionary cites Richard II, which it dates at 1593, for the earliest example of the usage. In this passage, a pale, gloomy Richard, facing the forces of Bolingbroke, tries to snap out of his despondency:

I had forgot myself; am I not king?
Awake, thou coward majesty! thou sleepest.
Is not the king’s name twenty thousand names?
Arm, arm, my name! a puny subject strikes
At thy great glory. Look not to the ground,
Ye favourites of a king: are we not high?

Help support the Grammarphobia Blog with your donation.
And check out our books about the English language.

Subscribe to the Blog by email

Enter your email address to subscribe to the Blog by email. If you are an old subscriber and not getting posts, please subscribe again.

Categories
English English language Etymology Expression Language Usage Word origin Writing

Good monsters and bad

Q: How did “monster” come to mean something extraordinarily good (“a monster of the violin”) as well as something extraordinarily bad (“a bloodthirsty monster”)?

A: Let’s begin by going back to the Middle Ages, when the first lexical monsters stalked the English language.

English borrowed “monster” from the Anglo-Norman and Middle French word monstre, but it ultimately comes from mōnstrum, classical Latin for an omen, a monstrous creature, a wicked person, or an atrocity.

When “monster” showed up in Middle English in the late 1300s, it could mean something extraordinary, unnatural, or ominous, as well as a mythical creature like Cerberus, the many-headed dog that guards the gates of the Underworld.

Chaucer uses the term both of those ways in the two earliest written examples for “monster” in the Oxford English Dictionary.

In Boece (circa 1374), his translation of the Roman philosopher Boethius’s De Consolatione Philosophiae, Chaucer uses “monster” to mean something extraordinarily worrisome:

“I vnderstonde þe felefolde colour and deceites of þilke merveylous monstre, Fortune” (“I understand the manifold wiles and deceits of this marvelous monster, Fortune”).

In “The Monk’s Tale” from the Canterbury Tales (circa 1386), Chaucer uses the term for such mythical creatures as the half-man, half-horse centaurs, and the half-woman, half-bird harpies:

“Was neuere wight sith that this world bigan / That slow so manye monstres as dide he” (“Was never such a man since this world began / That slew so many monsters as did he”). The reference is to Hercules, who killed centaurs and harpies and kidnapped Cerberus.

Over the years, the noun “monster” took on many related senses, including a sea monster or other huge creature (c. 1450); a cruel, wicked, or otherwise repulsive person (before 1505); and an ugly or deformed person or thing (1715).

However, the word lost much of its negative sense when used adjectivally in the 19th century to mean extraordinarily large. Early OED examples include “monster ballroom” (1837), “monster product” (1839), “Monster Meetings” (1843), and “monster organ” (1845).

We assume that this gigantic sense of the adjective inspired the use of “monster” in the 20th century as both a noun and an adjective for hugely successful people or things, and for people with a great amount of knowledge or talent.

In the early 20th century, writers began using “monster” colloquially to mean remarkably successful, profitable, or good. The first Oxford citation is from the Sept. 22, 1912, issue of the Sunday Times-Tribune (Waterloo, IA):

“New Plays, new specialties and new people will add to the big show’s drawing power and the prospects for a monster week are bright.” (The word “monster” here is a noun being used attributively—that is, as an adjective.)

In the mid-20th century, according to the Random House Historical Dictionary of American Slang, writers began using the noun positively to mean “a formidable aircraft or automobile.”

The first two examples in the slang dictionary are from Aviation Cadet (1955), a young-adult novel by Joseph Archibald: “We’re really throwing the iron monsters around now” … “We thought cockpit procedure on the monsters was confusing.”

In less than a decade, the noun took on the sense of a remarkable or successful person or thing. The earliest Random House example is from an April 6, 1968, interview in Rolling Stone:

“Of course, man, she’s a monster. She’s like the best of that type of singer.” In the interview, the guitarist Mike Bloomfield is describing Aretha Franklin.

In that same interview, Bloomfield uses “monster” adjectivally: “When I was around fifteen I was a monster rock guitar player.”

And in the mid-1980s, according to Random House, it came to mean “a person having formidable knowledge or skill,” as in this 1986 example from the ABC television sitcom Head of the Class: “Darlene—a speech and debate monster.”

Today, the word “monster” can refer to all of the above: a mythical creature, a threatening force, something unusually large, someone extraordinarily wicked, a great success, a remarkable talent, and so on. It all depends on how it’s used.

Help support the Grammarphobia Blog with your donation.
And check out our books about the English language.

Subscribe to the Blog by email

Enter your email address to subscribe to the Blog by email. If you are an old subscriber and not getting posts, please subscribe again.

Categories
English English language Etymology Expression Grammar Language Usage Writing

When ‘should’ means ‘would’

Q: In re-watching Downton Abbey, I’ve noticed that “should” is used in many places where I’d use “would.” For example, “If I were you, I should keep my mouth shut.” It’s very confusing at first to figure out what is meant. How did this usage evolve, and is it still heard in England?

A: We didn’t see that specific example in a search of Downton Abbey transcripts, but here’s a similar use of “should” by Violet, Dowager Countess of Grantham, from season 4, episode 3, of the TV series:

“If I were to search for logic, I should not look for it among the English upper class.”

In that example, “should” is used as an auxiliary verb to introduce a hypothetical future action, a usage that’s now generally expressed with “would” on both sides of the Atlantic.

The use of “should” for “would” can be confusing because “should” is commonly used to mean “ought to,” and “would” to mean “might.”

Why did Maggie Smith, who played the Dowager Countess in Downton Abbey, use “should” in that episode, which is set in the early 1920s?

To answer that, we’ll have to look at an old rule that drew a strict line between the use of “shall” and “will.” Here’s how we describe the rule in a 2011 post:

  • When expressing a future tense, use “shall” with the first person (“I” and “we”) and “will” with the second and third persons (“you,” “he,” “she,” “they,” etc.).
  • When expressing determination, permission, or obligation, use “will” with the first person and “shall” with the second and third persons.

As we note in that post, this so-called traditional rule has been observed more often in the UK than in the US, and the British haven’t been very observant.

John Wallis, an English clergyman and mathematician, introduced the rule in the 17th century in Grammatica Linguae Anglicanae, an English grammar book written (believe it or not) in Latin:

“In primis personis, shall simpliciter praedicentis est; will, quasi promittentis aut minantis / In secundis et tertiis personis, shall promittentis est aut minantis; will simpliciter praedicentis” (“In the first person, shall is simply for predicting, while will is for promising or threatening. / In the second and third persons, shall is for promising or threatening, while will is simply for predicting”).

Wallis applied the rule only to “shall” and “will,” but later usage writers broadened it to require “should” in the first person for future, conditional, and interrogative usages.

In A Dictionary of Modern English Usage, published in 1926, not long after the events portrayed in that Downton Abbey episode, Henry W. Fowler writes:

“Plain future or conditional statements & questions in the first person should have shall, should; the roman-type wills and woulds in the following examples are wrong.” Fowler’s no-no’s include “we will teach” … “we will soon be” … “will, I fear” … “we won’t get” … “would be a knave” … “would not” … “we would always get.”

So a dowager countess might very well have used “should” instead of “would” in the early 1920s.

However, Wallis’s old rule for using “shall” and “will,” as well as its expansion to “should” and “would,” has never been widely observed in the US or the UK—except for the use of “shall” in the first person in questions and legal documents.

As Jeremy Butterfield, editor of Fowler’s Dictionary of Modern English Usage (4th ed.), says in the latest (2015) version of the usage guide, “It is unlikely that the rule ever had its foundation in real usage, although it may have applied to some people some of the time.”

“The supposed rule is a dead letter in speech, and in most kinds of writing,” Butterfield writes. “It is broadly true that shall and should have largely retreated in the standard language even as used in England. In other English-speaking areas shall and should have been almost totally replaced by will and would, or by the reduced forms I’ll/we’ll. There is not much doubt that will will win, and shall shall lose, in the end.”

[Note, July 23, 2018. A reader in Australia writes: “I have never forgotten our English teacher’s explanation of the difference (London, England, late 1940s). You are on the beach and hear a voice calling out. By paying careful attention to the grammar, you can decide whether or not to enter the surf and risk your own life! ‘I shall drown! And no-one will save me!’ is a plea to be rescued. ‘I will drown, and no-one shall save me!’ is an announcement of firm intent to commit suicide!”

We’ve often come across this old memory aid, which was widely published in the 19th and early 20th centuries. The earliest example we’ve found is more than 200 years old. It’s from a short “filler” item published in the Feb. 4, 1804, issue of the Boston Weekly Magazine: “Difference Between Shall and Will. A Frenchman tumbled overboard, and sang out: ‘I will drown, and nobody shall help me.’ The sailors told him ‘drown and be d—d.’  Had he said, ‘I shall drown, and nobody will help me,’ the sailors would have saved him.”]

Help support the Grammarphobia Blog with your donation.
And check out our books about the English language.

Subscribe to the Blog by email

Enter your email address to subscribe to the Blog by email. If you are an old subscriber and not getting posts, please subscribe again.

Categories
English English language Etymology Expression Language Spelling Usage Word origin

Is it ‘realtor’ or ‘Realtor’?

Q: Why is “realtor” often capitalized? It drives me crazy. It’s just a job description, like “chef” or “dog catcher.” What’s so special about realtors?

A: The term is often capitalized because it’s a registered trademark in the US for a member of the National Association of Realtors.

Most standard dictionaries capitalize the term, including the online Merriam-Webster and American Heritage dictionaries. One notable exception is Oxford Dictionaries online, which lowercases the term in its US version.

The Associated Press Stylebook capitalizes “Realtor,” but recommends using “real estate agent” instead unless “there is a reason to indicate that the individual is a member” of the association.

The New York Times Manual of Style and Usage capitalizes the term too, and says the “preferred generic terms are real estate agent and real estate broker.”

However, the Oxford English Dictionary, an etymological dictionary based on historical evidence, lowercases “realtor.”

The OED says it’s a “proprietary name” for a member of the association, but adds, “Also in extended use,” which we take to mean that “realtor” is also used as a general term for anyone who sells real estate.

Garner’s Modern English Usage (4th ed.) has this to say: “Few people seem to know about the trademark, and consequently in AmE [American English] the term is used indiscriminately of real-estate agents generally.”

As for the etymology, Charles N. Chadbourn, a real estate agent in Minneapolis, coined the term in a March 15, 1916, article in the National Real Estate Journal, according to the OED.

Chadbourn, a member of the National Association of Real Estate Boards (predecessor of the National Association of Realtors), proposed  “that the National Association adopt and confer upon its members, dealers in realty, the title of realtor (accented on the first syllable).” And don’t misplace the “l”; it’s REAL-ter, not REE-luh-ter.

Interestingly, the term is lowercased there by Chadbourn, as well as in three of the other four examples for the usage in the OED. With language authorities divided over whether to capitalize it or not, the decision is up to you or the style manual you follow.

As for us, we normally use the term “real estate agent” when we refer to someone who sells real estate, whether a member of the association or not. The term “realtor” strikes us as too puffed up.

Sinclair Lewis, whose 1922 novel Babbitt is cited in the OED, apparently felt the same way. Lewis describes George F. Babbitt as “nimble in the calling of selling houses for more than people could afford to pay.”

In the OED citation, Babbitt is quoted as saying, “we ought to insist that folks call us ‘realtors’ and not ‘real-estate men.’ Sounds more like a reg’lar profession.”

Help support the Grammarphobia Blog with your donation.
And check out our books about the English language.

Subscribe to the Blog by email

Enter your email address to subscribe to the Blog by email. If you are an old subscriber and not getting posts, please subscribe again.

Categories
English English language Etymology Expression Language Pronunciation Usage Word origin Writing

Cycles and sickles

Q: Why does “bicycle” rhyme with “pickle,” and “motorcycle” with “Michael”?

A: Yes, “motorcycle” does usually rhyme with “Michael,” which has inspired various naughty playground rhymes (like “Michael, Michael, motorcycle, / Turn the key and watch him pee”).

However, “motorcycle” also rhymes with “pickle” in several regional dialects, according to the Dictionary of American Regional English.

DARE has examples from Appalachia and the Gulf region, including contributions from Kentucky, Tennessee, and Pennsylvania.

One example cites a lyric from “The Motor Cycle Song,” quoted in This Is the Arlo Guthrie Book (1969): “I don’t want a pick-le / Just want to ride on my mo-tor-sick-le.”

However, “motorcycle” (as well as “monocycle” and “unicycle”) normally rhymes with “Michael,” while “bicycle” (like “tricycle”) rhymes with “pickle.”

So why does the “y” in those four-syllable words sound like the long “i” of “sigh,” while the “y” in the three-syllable words sounds like the short “i” of “sick”?

This has to do with the way those syllables are stressed. A vowel is often pronounced one way in a stressed syllable and another way in an unstressed syllable.

The “y” of “bicycle,” for example, is unstressed and pronounced as a short, or reduced, vowel. But the word “cycle” itself has a “y” that’s stressed and pronounced as a long vowel. Similarly, the “y” of “motorcycle” has a secondary stress and a long pronunciation.

(By the way, “motorcycle”—like “monocycle” and “unicycle”—is made up of two trochees. A trochee is a metrical foot consisting of a stressed syllable followed by an unstressed one.)

As for the etymology, “cycle” first appeared in the 14th century, according to the Oxford English Dictionary. It originally meant a recurring period of time, such as a “lunar cycle” or a “solar cycle,” and it ultimately comes from the words for “circle” in classical Latin (cyclus) and Greek (κύκλος).

The noun took on the sense of a pedal-powered, wheeled vehicle in the 1870s, when it began to be used as a short form of the somewhat older words “bicycle,” “tricycle,” “monocycle,” and “unicycle.”

Earlier in the century, the term “velocipede” was used for a lightweight wheeled vehicle propelled by the rider. (As the OED comments, it was also called a “bone-shaker.”)

Oxford’s earliest example of “velocipede” is from a June 19, 1818, entry in the diary of William Sewall:

“Then I went to the circus and rode on the velocipede, which is a new machine.” (The diary begins in Maine, Sewall’s birthplace, and ends in Illinois.)

The dictionary’s earliest example for “bicycle” and “tricycle” is from the Sept. 7, 1868, issue of the Daily News (London): “Bysicles and trysicles which we saw in the Champs Elysées and Bois de Boulogne this summer.”

The words “monocycle” and “unicycle” showed up the following year in The Velocipede: Its History, Varieties, and Practice, an 1869 book by J. T. Goddard:

“A New York mechanic has devised a monocycle or single machine, which consists of a wheel eight feet in diameter, with a tire six inches wide” … “Hemming’s Unicycle or ‘Flying Yankee Velocipede.’ ” (We’ve expanded the first OED citation.)

The short use of “cycle” to mean a pedal-powered vehicle showed up the next year in “The Natural History of Bicycles,” an article in the February 1870 issue of Belgravia, a London magazine:

“Another idea for a monocycle (which, by the way, might be called a ‘cycle’ at once, for shortness) was to make a gigantic wheel, some twelve feet in diameter, with cranks on each side of the axle, and short stilts attached to these, to be worked by the rider’s feet.” (Again, we’ve expanded the OED citation.)

Later that same year, in August 1870, this passage appeared in the journal English Mechanic and Mirror of Science:

“I have never yet seen a bicycle, tricycle, or any other kind of cycle … which did not completely use up the whole muscular energy of the most muscular of muscular Christians.”

Finally, the first Oxford example for “motorcycle” is from the Atlanta Constitution, June 17, 1894:

“Some inventive genius with more activity in his brain than in his legs, has devised a cycle which appears to meet the utmost requirements of pure laziness. It is called the motor cycle and the propelling power is produced by coal oil.”

Help support the Grammarphobia Blog with your donation.
And check out our books about the English language.

Subscribe to the Blog by email

Enter your email address to subscribe to the Blog by email. If you are an old subscriber and not getting posts, please subscribe again.

Categories
English English language Etymology Expression Grammar Language Linguistics Phrase origin Usage Word origin Writing

‘More’ or ‘-er’? ‘Most’ or ‘-est’?

Q: Is there a rule for when to use “more” and “most” to form comparatives and superlatives, and when to use “er” and “est”? Why do we have two ways to do this?

A: There’s no “rule” about using “more” and “most” versus “-er” and “-est” to express the comparative and superlative. But there are some common conventions.

With “most adjectives and adverbs of more than one syllable, and with all those of more than two syllables,” the Oxford English Dictionary says, “the normal mode” of forming the comparative and superlative is by using “more” and “most.”

A few one-syllable words (like “real,” “right,” “wrong,” and “just”) also normally form comparatives and superlatives with “more” and “most” instead of with “-er” and “-est” suffixes, according to the OED.

The dictionary adds that “more” is also sometimes used with words of one or two syllables that would normally have “-er” comparatives, like “busy,” “high,” “slow,” “true,” and so on. Why? Here’s how Oxford explains it:

“This form is often now used either for special emphasis or clearness, or to preserve a balance of phrase with other comparatives with ‘more,’ or to modify the whole predicate rather than the single adjective or adverb, especially when followed by than.”

So we might choose “much more humble” instead of “much humbler.” Or we might say “so-and-so’s voice was more quiet but no less threatening.” Or “that’s more true than false.” Or even “his feet are more big than ungainly.”

The OED offers additional details about the the use of the “-er” and “-est” suffixes with adjectives and adverbs.

In modern English, the dictionary says, “the comparatives in -er are almost restricted to adjectives of one or two syllables,” while longer adjectives as well as two-syllable adjectives not ending in “-ly” or “-y” form the comparative “by means of the adverb more.”

The same goes for the “-est” suffix, which is used similarly to form the superlative of adjectives (Oxford points to its “-er” comparative entry for the “present usage” of the “-est” superlative).

As for the use of “-er” and “-est” with adverbs, those that have the same form as corresponding adjectives (“hard,” “fast,” “close,” etc.) chiefly form the comparative and superlative with “-er” and “-est,” while adverbs that end in “-ly” form the comparative with “more” and the superlative with “most.”

There are quite a few exceptions, of course. For a more comprehensive guide to how the comparative and superlative are expressed in English today, check out Jeremy Butterfield’s entry for “-er and -est, more and most” in Fowler’s Dictionary of Modern English Usage (4th ed.).

How did we end up with two ways to express the comparative and superlative in English? In a 2008 post, we discuss the etymology of “more” and “most” as well as the history of the suffixes “-er” and “-est.”

As we say in that post, the “-er” and “-est” suffixes have been used to make comparisons since the earliest days of English, and it’s a practice handed down from ancient Indo-European.

The Old English endings were originally spelled differently than they are today: -ra for the comparative, and -ost (sometimes -est) for the superlative.

Taking the word “old” as an example, the Old English forms were eald (“old”), yldra (“older”), yldest (“oldest”). And taking “hard” as another, the forms were heard (“hard”), heardra (“harder”), heardost (“hardest”).

Meanwhile, there was another set of Old English words: micel (meaning “great” or “big”), mara (“more”), and maest (“most”).

While “more” and “most” (or their ancestors) were around since the earliest days of English, it wasn’t until the early 1200s that we began using them as adverbs to modify adjectives and other adverbs in order to form comparatives and superlatives—that is, to do the job of the “-er” and “-est” suffixes.

For a few centuries, usage was all over the place. In fact, it wasn’t uncommon for even one-syllable words to be used with “more” and “most,” according to The Origins and Development of the English Language, by Thomas Pyles and John Algeo. The authors cite the frequent use of phrases like “more near,” “more fast,” “most poor,” and “most foul.”

And multi-syllable words were once used with “-er” and “-est,” like “eminenter,” “impudentest,” and “beautifullest.”

Pyles and Algeo say there were even “a good many instances of double comparison, like more fittermore better, more fairer, most worst, most stillest, and (probably the best-known example) most unkindest.”

Help support the Grammarphobia Blog with your donation.
And check out our books about the English language.

Subscribe to the Blog by email

Enter your email address to subscribe to the Blog by email. If you are an old subscriber and not getting posts, please subscribe again.

Categories
English English language Etymology Expression Language Usage Word origin Writing

Is that an “alum” on your bib?

Q: I am making some surprise bibs for a friend who is not sure of the baby’s sex. I want to put my friend’s college logo on the bib and write something along the lines of “Future (insert Logo here) Alumni.” I know “alumni” is plural and “alum” is slang, but I am not sure which word to use for a single person of unknown gender. My friend is a grammar geek and will not put her child in something with improper English.

A: If you don’t know whether the bib is for a future “alumnus” (boy) or “alumna” (girl), we’d recommend using “alum.”

Most standard dictionaries describe “alum” as “informal”—that is, suitable for relaxed or conversational usage. The term is increasingly being used, especially in American English, to describe a graduate of either sex.

It strikes us as just the right word for a baby’s bib, though we wouldn’t recommend it for a scholarly paper. If you think of “alum” as slangy, however, why not use “graduate” or the informal “grad”?

The Merriam-Webster Online Dictionary has a usage note entitled “Is it acceptable to use alum for alumnus or alumna?”

“Traditionally,” the dictionary says, “the word alumnus has been used to refer to a single male, whereas alumna has been used for a single woman. Initially the plural forms were alumni to refer to multiple men (or multiple men and women) and alumnae for multiple women.”

A little over a hundred years ago, according to M-W, “the shortened form of alum began to be used to describe a graduate or past attendee of either gender. Although many people feel that alum is informal, it is in increasing use, and we appear to be moving toward a greater acceptance of the word. The plural of alum is alums.”

As for the etymology, English borrowed “alumnus” and “alumna” from classical Latin, where an alumnus was a foster son, male child, protégé, ward, or pupil, and an alumna was its feminine counterpart.

When the two terms first appeared in English in the early 1600s, they referred to male and female students, not to graduates.

The earliest example for “alumnus” in the Oxford English Dictionary is from a 1602 religious tract by the English writer Anthony Copley: “Neither was I an Alumnus of the Colledge, being the Popes pensioner.”

The dictionary’s first example for “alumna” is from The Treasure of Vowed Chastity in Secular Persons, John Wilson’s 1621 translation from the Italian of a work by two Jesuit theologians, Leonardus Lessius and Fulvius Androtius:

“Take none of the younger sort of widowes, &c. which is meant that they should not be admitted into the function or ministery of diaconisses, or into the number of the Alumnae or Pupills of the Church.” (The plural is used here and in several other Oxford citations).

The first OED example for “alumnus” as a graduate or former student is from the Nov. 20, 1800, issue of the Maryland Gazette: “At the same time Messrs. Charles Alexander, … John Shaw and Carlisle F. Whiling, alumni of St. John’s college, were admitted to the degree of master of arts.”

The dictionary’s first citation for “alumna” as a graduate or ex-student is from the October 1843 issue of the Knickerbocker, a New York monthly magazine: “So favorably are we impressed with these ‘exercises’ of the alumnæ of the Albany Female Academy.”

Interestingly, a short form of “alumnus” and “alumna” showed up in the late 1600s, meaning a foster child, ward, protégé, or charge, but that sense is now considered obsolete or rare.

The only OED example is from a June 21, 1683, letter by John Eliot, a Puritan missionary: “Your hungry alumns do still cry unto your honour for the milk of the word.”

The first OED citation for a short form used to mean a graduate is from an 1877 speech by the Scottish botanist John Hutton Balfour.

At a Swedish ceremony marking the 400th anniversary of the University of Uppsala, he expressed hope that “Sweden may continue to send forth many alumns who shall do credit to her great Educational Institutions.”

Finally, the earliest Oxford citation for “alum” with the usual contemporary spelling is from the Dec. 13, 1928, issue of the Chicago Daily Tribune: “The local Harvard ‘alums’ have a number of parties in the incipient stage of planning.”

Help support the Grammarphobia Blog with your donation.
And check out our books about the English language.

Subscribe to the Blog by email

Enter your email address to subscribe to the Blog by email. If you are an old subscriber and not getting posts, please subscribe again.

Categories
English English language Etymology Expression Grammar Language Phrase origin Usage Word origin Writing

When ‘nor’ means ‘neither’

Q: Will you please address the use of “nor” in Shakespeare? Sometimes it differs from modern usage (“Of hot and cold, he was nor sad nor merry,” Antony & Cleopatra), and sometimes not (“He swore, had neither motion, guard, nor eye,” Hamlet).

A: You’ve spotted a construction that’s rare today—the poetic use of “nor” to mean “neither.”

This is sometimes seen in older poetry and drama, with “nor” replacing “neither” at the beginning of a series. So instead of writing “neither X nor Y,” a Shakespeare or a Dryden or a Pope might have written “nor X nor Y.”

As you noticed in Antony and Cleopatra (circa 1607) and Hamlet (c. 1600), Shakespeare might have used “nor” or “neither” at the beginning of a series—a choice undoubtedly determined by rhyme or reason.

The “nor” usage showed up in English writing around the beginning of the 16th century but is now rare, according to the Oxford English Dictionary. It’s usually found in the construction “nor — nor —” and is chiefly poetic, the dictionary says.

The earliest Oxford example is from Scotland, and the use is official rather than poetic:

“Nor ȝitt [yet] at the Sowth Loch nor yitt [yet] the North Loch.” (The phrase “nor yet” here means “and also not.” This passage, dated 1499-1500, was published in 1869 in Extracts From the Records of the Burgh of Edinburgh.)

All of the dictionary’s subsequent examples are from poetry or drama. Here’s a sampling, century by century.

1558: “Mischief close in keele doth growe, / Nor might of men can helpe, nor water floodes that on they throwe.” (From Thomas Phaer’s translation of Virgil’s Aeneid.)

1697: “Nor Bits nor Bridles can his Rage restrain.” (From John Dryden’s translation of Virgil’s Georgics.)

1726: “Now let our compact made / Be nor by signal nor by word betray’d.” (From Alexander Pope’s translation of Homer’s Odyssey.)

1800: “Nor shapes of men nor beasts we ken.” (From The Rime of the Ancient Mariner, by Samuel Taylor Coleridge.)

1913: “Nor God nor Daemon can undo the done, / Unsight the seen.” (From Thomas Hardy’s poem “To Meet, or Otherwise.”)

The most recent OED example is from Untitled Subjects (1969), a collection of poems by Richard Howard: “your only troth was plighted to Lady Laudanum, / to whom nor gout nor Paris could make you untrue.”

In a 2017 post, we discussed the use of the adverbs “neither” and “either” to introduce a series of more than two items, as in these examples from Shakespeare:

“You know neither me, yourselves nor any thing” (Coriolanus, c. 1605-08) … “Thou hast neither heate, affection, limbe, nor beautie” (Measure for Measure, c. 1604) … “They say there is divinity in odd numbers, either in nativity, chance, or death” (The Merry Wives of Windsor, c. 1597).

As we remarked in 2017, The Cambridge Grammar of the English Language says that “neither” and “either” can be used “in multiple as well as the more common binary coordination.”

There’s a similar explanation in the OED. It says that “following a word, phrase, or clause which is negated with neither,” the conjunction “nor” is “used before the second or further of two or more alternatives, normally to negate each.”

Help support the Grammarphobia Blog with your donation.
And check out our books about the English language.

Subscribe to the Blog by email

Enter your email address to subscribe to the Blog by email. If you are an old subscriber and not getting posts, please subscribe again.

Categories
English English language Etymology Expression Language Phrase origin Usage Word origin Writing

The El Niño problem

Q: Can you discuss the double-article problem that occurs when “the” is added to a phrase beginning with a definite article in another language? I am bothered to read or hear things like “The El Niño weather pattern is building.”

A: Technically, you’re right: “the” plus “El” does add up to “the the” when translated literally. But when a foreign phrase is established in English, the foreign article often isn’t translated literally—that is, interpreted as a separate instance of “the.”

In our opinion, “El Niño” has been assimilated and the Spanish term for the weather phenomenon can be used with an English definite article (as in “The El Niño weather pattern was blamed for the drought”). We’ll have more to say about this term later. First, a little background.

When a proper noun in a foreign language includes an article, the general practice is to use either the foreign article or “the,” but not both.

So in each set of examples below, you could use either version:

“They sang ‘La Marseillaise’ and marched to l’Arc de Triomphe” … “They sang the ‘Marseillaise’ and marched to the Arc de Triomphe.”

“La Costa Brava is our favorite region of Spain” … “The Costa Brava is our favorite region of Spain.”

“Don’t miss El Museo del Barrio” … “Don’t miss the Museo del Barrio.”

“We heard Die Meistersinger at Bayreuth” … “We heard the Meistersinger at Bayreuth.”

In cases like those, it would be redundant to use both the English and the foreign article (“the ‘La Marseillaise’ ” … “the Die Meistersinger,” and so on). This is because ordinarily the foreign article is interpreted as meaning “the,” even in an English context.

As The Chicago Manual of Style (17th ed.) says, “An initial the may be used if the definite article would appear in the original language.” Its examples include this one: “A history of the Comédie-Française has just appeared.”

There are exceptions, however, such as with the names of foreign newspapers. The Chicago Manual recommends that the foreign article should be retained if the newspaper’s name includes it.

The examples given include “El País” in Madrid, “Il Messagero” in Rome, “La Crónica de Hoy” in Mexico City, and “Al Akhbar” in Cairo. Each includes an article equivalent to “the.”

So, for instance, one would write, “He subscribes to Le Monde,” not “He subscribes to the Monde” (and definitely not “the Le Monde”).

And as we’ve said, a foreign article often isn’t treated as an actual article when a term of foreign origin becomes part of the English language. A few obvious examples are the Spanish names Los Angeles (literally, “the angels”) and Las Vegas (“the meadows”), and the French name for the game of lacrosse (“the stick”).

Despite their foreign derivations, and despite the literal meaning of los and las and la, these names have become thoroughly English and are used with English articles if an article is needed (“the Los Angeles Dodgers,” “the Las Vegas casino,” etc.).

We’re reminded of “the hoi polloi,” an expression that’s generally accepted by usage writers even though “hoi” represents “the.” The expression, whether two words or three, means “the masses” or “the common man” in English, and comes from οἱ πολλοί, classical Greek for “the many.”

As Bryan A. Garner writes in Garner’s Modern English Usage (4th ed.), “the three-word phrase has spread since about 1850, has become common, and ought to be accepted.”

Jeremy Butterfield, writing in the fourth edition of Fowler’s Modern English Usage, says “the hoi polloi” has “an impressive literary pedigree,” and leaving out the English article may be interpreted as “linguistic snobbery, misguided pedantry, or even unwholesome one-upmanship.”

Interestingly, the expression first showed up in English as “the οἱ πολλοὶ.” In a 1668 essay on dramatic poetry, John Dryden writes: “If by the people you understand the multitude, the οἱ πολλοὶ, it is no matter what they think; they are sometimes in the right, sometimes in the wrong: their judgment is a mere lottery.”

The first few examples for “hoi polloi” in the Oxford English Dictionary combine the English article with the original Greek phrase. The earliest OED citation for the Greek phrase written with the English alphabet also includes the English article.

In Gleanings in Europe by an American (1837), James Fenimore Cooper writes that a few great men lead every honorary institution “after which the oi polloi are enrolled as they can find interest.”

Getting back to the weather, we think “El Niño” has become so familiar in English that the foreign article has been absorbed into the name and has lost its separate sense of “the.”

We haven’t found much guidance on this issue, but judging from published examples, the “El” of “El Niño” is virtually never interpreted as a separate “the” in an English context. “El Niño” is treated as a phrasal noun for a weather phenomenon.

The New York Times’s stylebook is ambiguous on the subject, but it’s clear that Times editors don’t interpret “El” as “the.” We say this because Times articles have included phrases like “an El Niño,” “there was no El Niño,” “another El Niño,” “this new El Niño,” “a strong El Niño,” “the recent El Niño,” and “an El Niño year.”

If the Spanish article were being interpreted literally as “the,” those noun phrases would mean “a the Niño,” “there was no the Niño,” “another the Niño,” and so on. If there were any chance of such an interpretation, the editors would have omitted the foreign article (“a Niño,” “there was no Niño,” “another Niño”).

The OED, an etymological dictionary based on historical evidence, has several examples in which the Spanish article clearly isn’t being interpreted as a separate “the.” These include “the El Niño,” “the El Nino effect” and “the last five El Niños.” The dictionary doesn’t describe the examples as nonstandard or unusual in any way.

Oxford Dictionaries online, a standard dictionary, also cites many examples of the three-word expression without reservation: “the El Niño weather pattern” … “the El Niño phenomenon” … “the El Niño climatic conditions,” and so on.

Popular science publications, too, are willing to use English articles with these climate terms.

We’ve found examples in Scientific American of “the El Niño,” “the El Niño cycle,” “an El Niño,” “an El Niño event,” “the most recent El Niño,” “the last El Niño,” and “one of the strongest El Niños.”

And in Science magazine, you’ll find “the El Niño,” “an El Niño,” “this El Niño,” “the current El Niño,” “a strong El Niño event,” “the 1997–1998 El Niño,” and so on.

There’s less of this in academic journals, which tend to use “the El Niño Southern Oscillation” on first reference and “ENSO” on subsequent references. The longer technical name reflects the fact that El Niño results from an oscillation of atmospheric pressure in the tropical Pacific Ocean basin.

In Spanish, el niño means “the child.” The weather phenomenon is named for the Christ child, since its warmest sea surface temperatures off the South American coast are often recorded around Christmas.

Help support the Grammarphobia Blog with your donation.
And check out our books about the English language.

Subscribe to the Blog by email

Enter your email address to subscribe to the Blog by email. If you are an old subscriber and not getting posts, please subscribe again.

Categories
English English language Etymology Expression Language Phrase origin Usage Word origin Writing

An acid test—with real acid

Q: I see the phrase “acid test” often used during the World Cup competition in Russia to mean a crucial test for a team. Did it once refer to a test with real acid?

A: Not only did the phrase once mean a literal acid test—it still does, though the figurative sense is much more common.

When the term first showed up in writing in the mid-18th century, the Oxford English Dictionary says, it meant a “chemical test involving reaction with an acid.”

The dictionary’s earliest citation, which refers to boric acid, is from a 1759 essay by the English chemist Robert Dossie: “It has not the acid test of changing the colour of vegetable tinctures.”

However, the figurative use of the phrase to mean a crucial or conclusive test—such as the test facing a team at the World Cup—was inspired by the use of nitric acid to test gold for purity, according to the OED:

“The test for gold from which the figurative use developed typically involves making a mark on a touchstone with a piece of the metal in question and treating this mark with nitric acid, which dissolves other metals more readily than gold.”

As the dictionary explains, “The effect of the acid in dissolving the mark is compared with its effect on marks made by specimens of known gold content.”

Interestingly, the first Oxford example for “acid test” used to determine the purity of a precious metal refers to silver, not gold:

“The gentleman would then offer to bet $5 that the quarter was good, and would stand the acid test, which, as it was good silver [it] would of course do” (from the July 29, 1844, issue of a Philadelphia newspaper, the North American and Daily Advertiser).

The OED’s first gold example is from the Nov. 14, 1860, issue of a Wisconsin newspaper, the Monroe Sentinel: “The outside film of gold, though less than the two hundred thousandth part of an inch in thickness, is yet enough to cover up the base metal, and protect it from the usual acid test.”

However, we’ve found an earlier gold example in the Chemist, a London journal, in which the word “nitric” precedes “acid test.”

In an Aug. 15, 1850, letter, William Griffiths, a goldsmith in Dublin, reports an example of “the alloying of metals so that they should present the appearance of gold and be capable of being apportioned, so as not only to resist the nitric acid test but to deceive the most experienced as to color and weight.”

The first OED citation for the figurative sense is from the Nov. 18, 1854, Columbia Reporter, a Wisconsin paper: “Twenty-four years of service demonstrates his ability to stand the acid test, as Gibson’s Soap Polish has done for over thirty years.”

Here’s a more recent example from Spectacles, Lorgnettes, & Monocles, a 1989 book by D. C. Davidson: “Even an expert would hesitate to distinguish 9 carat from 12 and 14 carat gold without resorting to an acid test.”

As for today, the website of the Gemological Institute of America has a description of the touchstone acid test for gold. And you can find many acid-testing kits on Amazon.com.

We’ll end with a July 2, 2018, headline from the Northern Echo, a regional daily in the English town of Darlington.

“World Cup 2018: England about to face their acid test.” (England passed the test, beating Colombia to reach the quarterfinals.)

Help support the Grammarphobia Blog with your donation.
And check out our books about the English language.

Subscribe to the Blog by email

Enter your email address to subscribe to the Blog by email. If you are an old subscriber and not getting posts, please subscribe again.

Categories
English English language Etymology Expression Language Usage Word origin Writing

Why apposite isn’t opposite

Q: I have to stop and think every time I come to the word “apposite.” It looks something like “opposite,” but it means pretty much the opposite. I’ll bet there’s an interesting etymology here.

A: You betcha. “Apposite” and “opposite” look alike because they’re related. The two adjectives have a common ancestor, ponere, classical Latin meaning to put or place, according to the Merriam-Webster Online Dictionary.

Adding the prefix ad- (toward) to ponere gave Latin apponere (to place near) and the past participle appositus (near or appropriate), which ultimately gave us “apposite.”

Adding the prefix ob- (against) to ponere gave Latin opponere (to place against) and the past participle oppositus (against or opposite), source of our word “opposite.”

In the late 1300s, the word “opposite” showed up in English as both a noun and an adjective.

As a noun, it meant the opposite side or region; as an adjective, it referred to being on the opposite or farther end of a line. Both senses reflected the meaning of the Latin past participle oppositus.

The earliest citation in the Oxford English Dictionary for the noun is from “The Knight’s Tale” in Chaucer’s Canterbury Tales (circa 1386):

“Estward ther stood a gate of marbul whit, / Westward right swich another in the opposit” (“Eastward there stood a gate of white marble, / Westward another just the same in the opposite”).

The dictionary’s first example of the adjective is from The Equatorie of the Planetis (circa 1392), an anonymous Middle English treatise describing the construction and use of an instrument for calculating the position of the planets:

“Procede in the same litel cercle to ward lettere E opposit to D.” (Some scholars believe the treatise may have been written by Chaucer, based on the handwriting, style, and dialect of the manuscript.)

When “apposite” appeared in English in the early 1600s, Oxford says, it meant “well put or applied; appropriate, suitable (to),” similar to the sense of the Latin past participle appositus.

The earliest OED example is from The Anatomy of Melancholy (1621), Robert Burton’s wide-ranging treatise on a malady widespread in Jacobean England:

“A most apposite remedy.” (The remedy here is moderate sexual activity, as opposed to “Immoderate Venus,” which is said to cause the blues.)

Today, “apposite” means pertinent, relevant, or appropriate—that is, apt. And “opposite” as a noun refers to someone or something totally different from someone or something else, while as an adjective it means facing, on the other side of, or of an entirely different kind.

Help support the Grammarphobia Blog with your donation.
And check out our books about the English language.

Subscribe to the Blog by email

Enter your email address to subscribe to the Blog by email. If you are an old subscriber and not getting posts, please subscribe again.

Categories
English English language Etymology Expression Language Phrase origin Usage Word origin Writing

She’s gonna raise Cain

Q: I just came across an old joke (but new to me): “Adam and Eve were the world’s first troublemakers. They raised Cain.” Which makes me wonder about the origin of the expression “raise Cain.”

A: The verb “raise” in this expression originally meant to conjure up something like a spirit or demon, a usage that’s been around since the Middle Ages.

In the 19th century, this conjuring sense of “raise” inspired the use of the verb in various figurative phrases meaning to cause trouble.

One of them, to “raise Cain,” an American expression first recorded in the 1830s, would literally mean to summon the spirit of the biblical killer of Abel.

The literal use of “raise” in its conjuring sense first appeared in writing in the late 14th century.

In this sense, the Oxford English Dictionary says, it means “to cause (a spirit, demon, ghost, etc.) to appear, esp. by means of incantations; to conjure up.”

The dictionary’s earliest example is from “The Yeoman’s Tale,” part of Geoffrey Chaucer’s Canterbury Tales (circa 1386):

“I haue yow told ynowe / To reyse a feend al looke he neuere so rowe” (“I have told you enough to raise a fiend, look he never so fierce”).

Spirits “raised” in 15th-century writings included “deuils” (devils), “the devull,” and a “nygramansour” (necromancer or sorcerer). And in the 16th, 17th, and 18th centuries, they included ghosts, shades (apparitions), assorted dead notables, and “Grisly Spectres” (Milton, Paradise Regain’d, 1671).

In the 19th century, as we said, this sense of “raise” became figurative, which brings us around to Cain. To “raise the devil” or “raise Cain” came to mean, in the words of the OED, “to create a disturbance; to cause trouble, uproar, or confusion.”

The dictionary’s earliest example of “raise Cain” is coincidentally a version of that old joke about Adam and Eve:

“Why have we every reason to believe that Adam and Eve were both rowdies? Because … they both raised Cain.” (From a St. Louis newspaper, the Daily Pennant, May 2, 1840.)

However, we found a variation on the joke in a newspaper published two years earlier: “Why was Eve the first Sugar Planter? D’ye give it up? Because she raised Cain.” (From the Sangamo Journal/Illinois State Journal, April 7, 1838.)

If the phrase was familiar enough to be used in jokes and puns, “raise Cain” had obviously been around in common usage before those examples were published.

We’ll cite a handful of later 19th-century examples from the OED:

“They will feel that they have been raising Cain and breaking things” (from an 1841 collection of comic pieces, Short Patent Sermons, by “Dow, Junior,” the pen name of Elbridge Gerry Paige).

“Topsy would hold a perfect carnival of confusion … in short, as Miss Ophelia phrased it, ‘raising Cain’ generally” (from Harriet Beecher’s Stowe’s novel Uncle Tom’s Cabin, 1852).

“I expect Susy’s boys’ll be raising Cain round the house” (from Stowe’s novel Oldtown Folks, 1869).

“If I get the horrors, I’m a man that has lived rough, and I’ll raise Cain” (from Robert Louis Stevenson’s Treasure Island, 1883).

As for “raise the devil,” the OED’s earliest confirmed example is from 1841, but we found this slightly earlier usage in a Virginia newspaper:

“Wm. Colson came up, and says, ‘Don’t talk so loud, for there are a great many Albany people on board, and if they find out that I’m engaged in this business, they will raise the devil with me’ ” (from court testimony in a fraud case, published in the Richmond Enquirer, Nov. 3, 1840).

Other satanic specters were apt to be “raised” in the troublemaking sense. Some related expressions, and the earliest dates we’ve found, include “raise Ned” (1845, a euphemistic reference to the devil), “raise mischief” (1840, another euphemism for the devil), and “raise Hell” (1803).

On that last expression, the OED has this fascinating aside: “The slogan ‘Kansas should raise less corn and more hell’ is attributed to Mrs. Mary Ellen Lease (1853–1933) but proof is lacking.”

We’ll end with a musical rendition of “raise Cain.” It’s from Ella Fitzgerald’s recording of the 1932 song “I’ll Be Hard to Handle,” with music by Jerome Kern and lyrics by Bernard Dougall. Here are a couple of stanzas:

I’ll be hard to handle
I’m telling you plain
Just be a dear
And scram out of here
I’m gonna raise Cain.

I’ll be hard to handle
I’m no ball and chain
I’ll find some means
To call the Marines
I’m gonna raise Cain.

Help support the Grammarphobia Blog with your donation.
And check out our books about the English language.

Subscribe to the Blog by email

Enter your email address to subscribe to the Blog by email. If you are an old subscriber and not getting posts, please subscribe again.

Categories
English English language Etymology Expression Language Usage Word origin Writing

A picayune question

Q: Why is something small and insignificant called “picayune”? And what is the word doing in the name of a New Orleans newspaper?

A: The word “picayune” comes from picaillon, a southern French regional term for a small coin of foreign origin, according to the Oxford English Dictionary.

The OED says the French regionalism is derived from picalhon, an Occitan term for a 17th-century copper coin that was minted in the Savoy and Piedmont regions of southern Europe, and that inspired similar cheaply made coins elsewhere in Europe.

When “picayune,” an Anglicized version of picaillon, showed up in Louisiana in the early 1800s, it was a noun that referred to a Spanish medio real, or half real, a coin worth a little more than six cents, and later to a US nickel, according to the dictionary.

The first Oxford example for “picayune” is from a Nov. 4, 1805, entry in the journal of the Philadelphia antiquarian John Fanning Watson: One can’t buy anything [at New Orleans] for less than a six cent piece, called a picayune.”

We suspect that French speakers in Louisiana may have used picaillon earlier for the coin, but we haven’t found written evidence to support this. (The Louisiana region was variously ruled by France and Spain before becoming an American territory in 1803. Spanish coins were legal tender in the US from 1793 to 1857.)

In a few decades, the OED says, “picayune” was being used as an adjective meaning of “of little value; paltry, petty, trifling; unimportant, trivial; mean; contemptible.”

This example is from an 1837 congressional debate: “The hon. Senator from Kentucky … by way of ridicule, calls this a ‘picayune bill.’ ” (From the Congressional Globe, which recorded debates of the 23rd through 42nd Congresses, 1833-’73.)

A year later, the noun came to mean a small amount of something, as in this Oxford example from the February 1838 issue of Godey’s Lady’s Book, a Philadelphia magazine: “I have nothing, not one sous—not a picayune to give her!”

And in the early 20th century, “picayune” took on the sense of a “worthless or contemptible person.” The first OED citation is from a 1903 issue of Scribner’s Magazine: “A pack of jealous picayunes, who bickered while the army starved.”

Why does the word “picayune” appear in the name of the Times-Picayune, the New Orleans newspaper? Because when it was founded in 1837, the Picayune (the paper’s name before it merged with the Times-Democrat in 1914) cost one picayune, or Spanish half real.

Help support the Grammarphobia Blog with your donation.
And check out our books about the English language.

Subscribe to the Blog by email

Enter your email address to subscribe to the Blog by email. If you are an old subscriber and not getting posts, please subscribe again.

Categories
English English language Etymology Expression Language Phrase origin Usage Word origin Writing

Going Berserk with a capital ‘B’

Q: I was reading an old Mary Roberts Rinehart mystery, The Circular Staircase, when my eyes fell upon this passage: “I had gone Berserk, I think. I leaned over the stair-rail and fired again.” Why is “Berserk” capitalized?

A: The word used to be capitalized in English because it was originally a proper noun. The “Berserks” were legendary Norse warriors who went into battle in a wild, murderous frenzy, according to Scandinavian mythology.

The novel you mention was published in 1908, over a century ago, and “berserk” was often capitalized in those days.

The word came into English as a noun in the early 19th century, but by the late 1800s, it had become an adjective, as it is in that quotation (“I had gone Berserk”).

English acquired “berserk” from Old Icelandic, where berserkr is a singular noun, berserker the plural, and berserk the accusative (the form used for a direct object), according to the Oxford English Dictionary.

Sir Walter Scott introduced the word into English in 1814 as “Berserkar,” the singular noun for the warrior, and “Berserkir,” the plural. (Later, Scott used “Berserkars” for the plural, and other 19th-century authors shortened these nouns to “Berserk” and “Berserks.”)

In an article summarizing the Eyrbyggja Saga, a 13th-century work in Old Icelandic, Scott describes the warriors this way:

“Berserkir, men who, by moral or physical excitation of some kind or other, were wont to work themselves into a state of frenzy, during which they achieved deeds passing human strength, and rushed, without sense of danger or feeling of pain, upon every species of danger that could be opposed to them.”

The article (published in the anthology Illustrations of Northern Antiquities, Vol. I) uses both the singular (“a haughty, fiery, and robust damsel, well qualified to captivate the heart of a Berserkar”) and the plural (“the two Berserkir”) several times.

The OED cites Gudbrand Vigfusson and Johan Fritzner, Old Norse and Old Icelandic scholars, as saying the original meaning of “berserk” was probably bear-shirt or bear-coat.

(In Old Icelandic and Old Norse, serkr means “coat” or “shirt”; it’s a cousin of an obsolete English word for a shirt, “sark.”)

Some etymologists have disagreed with this explanation, arguing that the ber– element in Icelandic was for “bare,” not “bear,” suggesting that the warriors went into battle without any armor, either bare-chested or wearing only their shirts.

Scott himself probably contributed to this belief. In his novel The Pirate (1822), he mentions “those ancient champions, those Berserkars,” and in a note to the 1831 edition explains that they were “so called from fighting without armour.”

We may never know which etymology is correct. What we do know is that the Berserks showed up in many of the medieval Icelandic and Norse narratives, from the 9th century onwards, that describe events in ancient Scandinavian mythology.

This line from a 9th-century Old Norse poem by Thornbjorn Hornklofi is one of the earliest known examples in writing: “Grenjuðu berserkir, guðr vas þeim á sinnum” (“Berserks bellowed; battle was under way for them”).

The OED defines the noun “berserk” as “a wild Norse warrior of great strength and ferocious courage, who fought on the battle-field with a frenzied fury known as the ‘berserker rage’; often a lawless bravo or freebooter.”

Today, Oxford adds, the word is used as an adjective meaning “frenzied, furiously or madly violent,” and it’s commonly found in the phrase “to go berserk.”

In addition to Scott, the OED cites a few other 19th-century authors who used long forms of the word: “Berserkers” (1837), “Berserkir-rage” (1839), and “bersarkar” (in the singular, 1861). Still later, the nouns “Berserk” and “Berserks” appeared.

The dictionary’s earliest use of the short form “berserk” is also its earliest use of the adjective. It’s from Charles Kingsley’s novel Yeast (1851): “Yelling, like Berserk fiends, among the frowning tombstones.”

Perhaps “berserk” was a household word with the Kingsleys. Charles’s younger brother, Henry Kingsley, used the adjective in his novel Silcote of Silcotes (1867): “With her kindly, uncontrollable vivacity, in the brisk winter air she became more ‘berserk’ as she went on.”

In many later appearances the adjective was still being capitalized, as in this OED example: “He … was filled with a Berserk rage and thirst for retribution.” (From James Hannington, First Bishop of Eastern Equatorial Africa, a biography written in 1886 by Edwin C. Dawson and published in 1887.)

In searches of historical newspaper databases, we found instances of “became Berserk” (1867) and “turn Berserk” (1877), but none with forms of the verb “go” until the 1890s.

The earliest example we’ve found (“going berserk”) is from the April 6, 1894, issue of the Aspen (CO) Daily Times:

“He never had the gold or diamond or colonial fever; instead of going berserk, he evidently preferred a frock coat and patent leathers.” (From an anonymous short story, “The Panic.” Though the story is credited to the London Illustrated News, we failed to find it there.)

The following year this example appeared in “The Child of Calamity,” a short story by Rudyard Kipling: “Then he went Berserk before our amazed eyes.”

(The story was published in March and April 1895 in several newspapers in Australia and the US. The Idler, a British magazine, published it under a different title, “My Sunday at Home,” in April 1895.)

We also found many uses of “go” plus “berserk” in newspapers published in the early 1900s. Note the differing capitalization styles:

“It would be dangerous to allow the smaller settlers to go berserk before the board, and meet the serried ranks of officialdom.” (The Sydney Morning Herald, Aug. 15, 1905.)

“We knew him as a dashing and fearless old campaigner, one who had gone Berserk many a time to rescue the gallant Lat Sahib he fought under.” (This passage appeared in another Australian newspaper, the Register, Adelaide, on Oct. 14, 1905. It’s from a serialized novel, Tales of Sahib Land, by the Anglo-Indian writer F. D’A. C. De L’Isle.)

We found many other pre-World War I examples illustrating the different capitalization styles, including “go Berserk” (1908), “went ‘berserk’ ” (1908), “went Berserk” (1909), and so on.

Even during WWI, some authors were still capitalizing the word, as in this OED example from Rudyard Kipling’s novel A Diversity of Creatures (1917):

“ ‘You went Berserk. I’ve read all about it in Hypatia.’ ” (Kipling is apparently referring to Charles Kingsley’s novel about Hypatia, a philosopher in Alexandria, Egypt, in the late 4th and early 5th centuries. We haven’t found “berserk” in the novel, though Kingsley uses it in at least two others.)

Before long, however, the adjective became established in its lowercase form. The OED cites this headline from the Nov. 10, 1940, issue of the Chicago Tribune: “America goes berserk.”

The article below the headline comments upon “The recent addition of the word ‘berserk,’ as a synonym for crackpot behaviour, to the slang of the young and untutored. … American stenographers … are telling one another not to be ‘berserk.’ ”

But “berserk” was not a slang or “untutored” usage, as we’ve seen. And for generations it has continued to be used—as an adjective, lowercased, and mostly with the verb “go”—as standard English.

It has even retained its sense of violent frenzy, though the violence is milder today that it was in the days of the Vikings.

In tribute to the old sagas, we’ll conclude with a passage from Henry Wadsworth Longfellow’s poem “The Skeleton in Armor” (1842):

Many a wassail-bout
Wore the long Winter out;
Often our midnight shout
   Set the cocks crowing,
As we the Berserk’s tale
Measured in cups of ale,
Draining the oaken pail,
   Filled to o’erflowing.

Help support the Grammarphobia Blog with your donation.
And check out our books about the English language.

Subscribe to the Blog by email

Enter your email address to subscribe to the Blog by email. If you are an old subscriber and not getting posts, please subscribe again.

Categories
English English language Etymology Expression Language Usage Word origin Writing

Are you anxious or eager?

Q: In 2012, the two of you were divided over the use of “anxious” to mean “eager.” I’m eager to learn if you’re still at odds, and anxiously await an update.

A: Put your mind at rest. We both now agree that one can be “anxious” as well as “eager” to do something, though not all language mavens are ready to join us.

The naysayers reject a sentence like “We were anxious to see the new musical.” They believe “anxious” can be used only if there’s anxiety involved: “She’s anxious to see a cardiologist about the palpitations.”

As Bryan A. Garner writes in Garner’s Modern English Usage (4th ed.), “when no sense of uneasiness is attached to the situation, anxious isn’t the best word” and “it displaces a word that might traditionally have been considered its opposite—namely eager.”

However, the anxiety-free use of “anxious” to mean “eager” was well-established before American usage authorities began questioning the practice in the early 20th century.

English borrowed the adjective “anxious” from classical Latin, where anxius meant worried, disturbed, uneasy, and so on. But “anxious” began evolving soon after it showed up in English in the 16th century.

When it first appeared in writing, “anxious” referred to someone “experiencing worry or nervousness, typically about the future or something with an uncertain outcome,” according to the Oxford English Dictionary.

The earliest OED example is from Nicholas Lesse’s 1548 translation of a Latin treatise by the French theologian François Lambert: “Wherfore do we then endeuour oure selues to do anie thinge, wherefore are we so anxiouse & careful?”

In a couple of decades, however, “anxious” was being used before an infinitive to express a strong desire or eagerness to do something, sometimes with anxiety and sometimes without.

The first Oxford example for the new sense is from Actes and Monuments of Matters Most Speciall and Memorable, Happenyng in the Church (2nd ed., 1570), an ecclesiastical history by John Foxe.

In the citation, Prince Frederick III, the Elector of Saxony, is described as “very anxious and inquisitiue to heare the iudgementes of other, which were both aged, & learned.”

As far as we can tell, there’s no anxiety here. Although Frederick faced an important decision, Foxe earlier describes him as an easygoing man who “loued best quietnes & cōmon trāquilitie” and was “trustyng to hys owne iudgemēt.”

(After consulting Erasmus, the Prince decided to protect Martin Luther despite the opposition of Pope Leo X and Charles V, the Holy Roman Emperor.)

Merriam-Webster’s Dictionary of English Usage has many anxiety-less, infinitive examples in which respected writers use “anxious” to do something in the sense of “eager” to do it:

Lord Byron, in Canto XV of his poem Don Juan (1824), writes: “His manner was perhaps the more seductive, / Because he ne’er seem’d anxious to seduce.”

And in Omoo (1847), a semi-fictionalized South Seas memoir, Herman Melville writes that “the men looked hard at him, anxious to see what sort of looking ‘cove’ he was.”

Charles Darwin, in On the Origin of Species (1859), says, “I could give many facts, showing how anxious bees are to save time.”

And in Robert Louis Stevenson’s novel Treasure Island (1883), Jim Hawkins, the narrator, says “anxious as I was to tell them my story, I durst not interrupt.”

Merriam-Webster’s conclusion? “Anyone who says that careful writers do not use anxious in its ‘eager’ sense has simply not examined the available evidence.”

As far as we can tell, writers used “anxious” in place of “eager” for hundreds of years before anyone raised an eyebrow at the usage.

“The discovery that anxious must not be used to mean ‘eager’ seems to have been made in the U.S. in the early 20th century,” M-W says

The first usage guide to criticize the practice was Ambrose Bierce’s Write It Right (1909). But several years earlier, in July 1901, the language writer Alfred Ayres criticized it in “A Plea for Cultivating the English Language,” an article published in Harper’s Monthly Magazine.

Interestingly, British usage writers haven’t been troubled by the use of “anxious” for “eager.” Henry W. Fowler, in the first edition of A Dictionary of Modern English Usage (1926), describes it as a “natural” development and “almost universally current.”

The latest version of Fowler’s Dictionary of Modern English Usage (4th ed.), by Jeremy Butterfield, describes the use of “anxious” for “eager” as “historically well attested” and “absolutely standard.”

In recent years, American critics of the anxiety-free use of “anxious” with an infinitive have been coming around.

The American Heritage Dictionary of the English Language notes in its online edition: “In our 1999 survey of the Usage Panel, 47 percent approved of the sentence We are anxious to see the new show of British sculpture at the museum, whereas in 2014, this sentence was acceptable to 57 percent of panelists.”

“Although resistance to the use of anxious to mean eager is waning,” American Heritage cautions, “writers should be aware that there are still those who frown upon using the word in situations where no anxiety is present.”

Help support the Grammarphobia Blog with your donation.
And check out our books about the English language.

Subscribe to the Blog by email

Enter your email address to subscribe to the Blog by email. If you are an old subscriber and not getting posts, please subscribe again.

Categories
English English language Etymology Expression Language Usage Word origin Writing

Is ‘film’ classier than ‘movie’?

Q: When did the medium “film” become the “film” we watch? Did English speakers think “film” was a classier word for the art form than “movie”? As a Sam Shepherd character says in True West, “In this business we make Movies, American Movies. Leave the Films to the French.”

A: English speakers didn’t begin using “film” for a motion picture because they wanted an artier, Frenchified word than “movie.” In fact, this use of “film” showed up in English before “movie.” And we didn’t get “film” from French—the French got it from us. Here’s the story.

When “film” appeared in Old English (spelled filmen, filmin, fylmen, etc.), it meant a “thin layer or sheet of tissue in an animal or plant, or in a product of an animal or plant,” according to the Oxford English Dictionary.

The earliest OED example is from Bald’s Leechbook, a medical text believed written around 900: “Her sint tacn aheardodre lifre, ge on þam læppum & healocum & filmenum” (“Here are the symptoms of a liver hardened on the lobes and the recesses and the films”).

In the early 1600s, “film” took on the sense of a “very thin sheet of any substance,” according to the dictionary, and by the mid-1800s it came to mean “a thin layer of light-sensitive material, typically applied to photographic paper or plates and used to record a photographic image.”

The OED’s earliest example of “film” used in the photographic sense refers to the sheet of silver-plated copper used to make a daguerreotype image:

“We must separate carefully the chemical changes which iodide of silver undergoes in the sunbeam, from the mechanical changes which happen to the sensitive film” (from an 1840 issue of the London, Edinburgh, and Dublin Philosophical Magazine and Journal of Science).

By the late 1800s, the word “film” was being used for “a thin flexible strip of celluloid, plastic, etc., coated with light-sensitive emulsion, used in photography and cinematography to record a series of images.”

The first written use in the OED is from an 1895 Montgomery Ward catalog: “Roll Film, for 25 exposures.”

In the early 1900s, according to Oxford, “film” took on the sense you’re asking about: “a representation of a story or event recorded on film” and “shown as moving images in a cinema or (latterly) on television, video, the Internet, etc.”

The dictionary’s first citation (which we’ve expanded) is from the Jan. 21, 1905, issue of the Westminster Gazette (London):

“The plaintiff is an eminent Parisian surgeon, the defendants a firm who took cinematograph films of his operations. This he allowed them to do, so that he might get scientific records, but the films once obtained have been sold and even exhibited at country fairs.”

The earliest Oxford example of “movie” used in this sense appeared more than five years later. “I finally decided to have a look-in on some of the programs of vaudeville and movies” (from the May 22, 1910, issue of the Philadelphia Inquirer).

But before either “film” or “movie” appeared on the scene as a cinematic work, the terms “moving picture” and “motion picture” were used similarly.

The dictionary’s first “moving picture” citation, which we’ve expanded, is from an Oct. 3, 1896, letter written by Queen Victoria from Balmoral Castle in Scotland:

“At twelve went down to below the terrace, near the ballroom, and we were all photographed by Downey by the new cinematograph process, which makes moving pictures by winding off a reel of films.” William Downey’s moving picture of the Queen is available online.

Oxford’s earliest citation for “motion picture” used this way is from the November 1900 issue of Catholic World:

“That evening, during a reproduction of the Passion Play in motion pictures, a magnificent tenor, interspersing the pictured scenes with arias appropriate to the theme, was received with enthusiasm.”

(The OED has an 1891 citation for “motion picture,” but the term is used to mean a movie camera, not a movie.)

By the way, the colloquial term “flick” first appeared in The Square Emerald, a 1926 mystery by the English writer Edgar Wallace: “We’ll occupy the afternoon with a ‘flick.’ I love the movies—especially the romantic ones.”

The word “flicker,” used the same way, showed up in print a year later. But we suspect that it was around earlier in speech, and that “flick” was a shortening of “flicker.” Both apparently refer to the flickering appearance of old movies.

Now for the latecomer, the French use of film to mean an oeuvre cinématographique.

The earliest example in Le Trésor de la Langue Française, an etymological and historical dictionary of the French language, is from Histoire de l’Art du Cinéma des Origines à Nos Jours, a 1949 book by the cinema writer Georges Sadoul:

“L’ému et tendre Silence est d’or a été le meilleur film qu’ait dirigé René Clair depuis son départ de Paris” (“The moving and tender Silence Est d’Or was the best film directed by René Clair after his departure from Paris”).

When the French originally borrowed the term film from English in the late 1800s, it referred to a bande de pellicule (“strip of celluloid”) used to make photographs or motion pictures.

The French dictionary describes the use of the medium “film” for a work made from it as an example of metonymy, a figure of speech in which a word or phrase is used as a substitute for something it’s closely associated with

Help support the Grammarphobia Blog with your donation.
And check out our books about the English language.

Subscribe to the Blog by email

Enter your email address to subscribe to the Blog by email. If you are an old subscriber and not getting posts, please subscribe again.

Categories
English English language Etymology Grammar Language Usage Writing

Are families people or things?

Q: My work at a university involves writing about families. When referring to them, should I use “who” or “that”? For example, “families who eat together” vs. “families that eat together.” In other words, are families people or things?

A: “That” is our oldest and most flexible relative pronoun. It’s been used since the Middle Ages for both people and things. If in doubt, you can’t go wrong with “that.”

The relative pronoun “who,” on the other hand, is used exclusively for people and animals personified with personal names.

Grammatically speaking, the noun “family” (like “class,” “committee,” “orchestra,” “faculty,” and so on) is a thing, even though it’s made up of people. So your example should read “families that eat together.”

But even if one argues that the noun “family” implies people, “that” is an appropriate relative pronoun, since it can be used for both people and things.

Despite what many people think, there’s no foundation for the widespread belief that “that” should refer only to things and “who” only to people.

As Pat writes in her grammar book Woe Is I, “that has been used for people as well as inanimate things for some eight hundred years, and it’s standard English. The girl that married dear old Dad was Mom.”

The Oxford English Dictionary, an etymological dictionary based on historical evidence, has examples from Anglo-Saxon times for “that” (ðæt, ðet, and þhet in Old English) used as a relative pronoun for both people and things.

The dictionary’s earliest example of “that” in reference to a thing is from the Vespasian Psalter, an illuminated manuscript written around 825: “In bebode ðæt ðu bibude” (“In the command that thou commanded”).

The OED’s earliest example of “that” referring to a person is from the Lambeth Homilies, written around 1175: “Þes Mon þhet alihte from ierusalem in to ierico” (“This man that descended from Jerusalem into Jericho”).

And “that” has been used in both ways ever since.

A post we wrote in 2007 includes an excerpt from A Dictionary of Contemporary American Usage, by Bergen Evans and Cornelia Evans, about the history of the relative pronouns “that,” “who,” and “which.” It’s worth repeating:

That has been the standard relative pronoun for about eight hundred years and can be used in speaking of persons, animals, or things. Four hundred years ago, which became popular as a substitute for the relative that and was used for persons, animals, and things. Three hundred years ago, who also became popular as a relative. It was used in speaking of persons and animals but not of things. This left English with more relative pronouns than it has any use for. … Who may in time drive out that as a relative referring to persons, but it has not yet done so.”

As we said in 2007, you can undoubtedly find writers on grammar and usage who disagree with this conclusion, but we think it’s sound. We still do.

Help support the Grammarphobia Blog with your donation.
And check out our books about the English language.

Subscribe to the Blog by email

Enter your email address to subscribe to the Blog by email. If you are an old subscriber and not getting posts, please subscribe again.

Categories
English English language Expression Grammar Language Usage Writing

The prepositional subject

Q: I’m studying English in Japan, and I’m confused by the use of prepositional phrases as subjects, as in this example: “Across the field is the nearest way to the lake.” Is this a common usage? Can prepositional phrases be objects too?

A: A prepositional phrase can be the subject, object, or complement of a verb. This is a common construction in English, and often the verb is a form of “be,” as in your example. Here are a few more illustrations.

As subject: “Over the mantle is a good place for the mirror” … “From five to seven would be the best time.”

As object: “He directed between 50 and 60 movies” … “The project will take over a week.”

As complement: “A good place for the mirror is over the mantle” … “The best time would be from five to seven.”

(With a linking verb like “be,” a subject complement occupies the position of the object.)

The Cambridge Grammar of the English Language cites these examples, with “spent” as the verb: “Over a year was spent on this problem” (subject) … “I spent over a year here” (object).

Prepositional phrases can be modifiers, too. So they can act as adverbs (“Puffin lies on the bed” … “In the afternoon, Puffin naps”) or as adjectives (“The cat on the bed is Puffin” … “Naps in the afternoon are her favorite”).

In fact, a prepositional phrase “is by far the commonest type of postmodification in English,” according to A Comprehensive Grammar of the English Language, by Randolph Quirk et al.

The authors give many examples of such modifiers following nouns, including “the car outside the station,” “the road to Lincoln,” “this book on grammar,” “passengers on board the ship,” “action in case of fire,” “the house beyond the church,” and “two years before the war.”

In addition, the authors explain, prepositional phrases can complement a verb (“We were looking at his awful paintings“) or an adjective (“I’m sorry for his parents“).

Help support the Grammarphobia Blog with your donation.
And check out our books about the English language.

Subscribe to the Blog by email

Enter your email address to subscribe to the Blog by email. If you are an old subscriber and not getting posts, please subscribe again.

Categories
English English language Etymology Expression Language Usage Word origin Writing

‘Hogwash’ vs. ‘claptrap’

Q: I characterized a theory in an academic paper as “hogwash.” A well-read colleague thought “claptrap” might have been better. I can’t figure out a meaningful distinction. I suspect this would not be interesting to your readers, but just in case …

A: Yes, there isn’t much of a distinction between “hogwash” and “claptrap” these days, but we don’t think a discussion about their use in academia would be academic to our readers.

Although both “hogwash” and “claptrap” mean nonsense, “claptrap” may also suggest pretentiousness and insincerity to those aware of its etymology, while “hogwash” may still have a whiff of the barnyard for some.

When “hogwash,” the older of the two terms, showed up in the mid-1500s, according to the Oxford English Dictionary, it meant “kitchen refuse and scraps (esp. in liquid form) used as food for pigs.”

The earliest OED example is from Jacob’s Well, an anonymous Middle English sermon cycle, written around 1450, in which the human soul is likened to a foul well in need of cleansing. Here a humble nun is humiliated by other nuns in her convent:

“Þey in þe kechyn, for iape, pouryd on here hefd hoggyswasch” (“For a joke, they had poured hogwash on her in the kitchen”).

In the early 1600s, Oxford says, “hogwash” took on the sense of a “liquid for drinking that is of very poor quality, as cheap beer, wine, etc.”

The dictionary cites A New Description of Ireland (1610), by the English writer and soldier Barnabe Rich: “The very remembrance of that Hogges wash which they vse to sell for ij.d. the Wine quart, is able to distemper any mans braines.”

In the late 1800s, the word took on the modern meaning of “nonsense; esp. worthless, ridiculous, or nonsensical ideas, discourse, or writing.” The first OED example is from an article by Mark Twain in the June 1870 issue of Galaxy Magazine, a short-lived American monthly:

“I will remark, in the way of general information, that in California, that land of felicitous nomenclature, the literary name of this sort of stuff is ‘hogwash.’ ” (The “stuff” here is “sham sentimentality” in literature.)

However, the term “hogwash” hasn’t entirely escaped its porcine origins, especially for punning headline writers, as you can see from these examples:

“Swine intervention: California animal lovers call pig rescue a load of hogwash” (The Guardian, June 16, 2017), and “No hogwash: Pigs shut down Ky. highway after semi overturns” (the Daily Nonpareil, Council Bluffs, IA, Aug. 9, 2017).

As for “claptrap,” it originated in the early 1700s as theatrical jargon for a “trick or device to catch applause; an expression designed to elicit applause,” the OED says. The dictionary’s earliest example, which we’ve expanded, is from The Universal Etymological English Dictionary (Vol. 2, 1727), by Nathan Bailey:

“A CLAP Trap: A name given to the rant and rhimes that dramatick poets, to please the actors, let them go off with; as much as to say, a trap to catch a clap by way of applause from the spectators at a play.”

By the early 1800s, “claptrap” was being used to mean catchy language or cheap, showy sentiment, as in this OED citation, which we’ve also expanded, from Byron’s satirical poem Don Juan (Canto II, 1819): “I hate all mystery, and that air / Of clap-trap, which your recent poets prize.”

And by the late 1800s, the word had acquired its modern meaning of nonsense, as in this Oxford example from Benjamin Disraeli’s 1880 novel Endymion: “He disdained all cant and clap-trap.”

You’re right that most people would see no meaningful distinction between “hogwash” and “claptrap.” But a sensitive academic with the OED handy might be ticked off more by one than the other, though we can’t imagine which term would be more upsetting.

Help support the Grammarphobia Blog with your donation.
And check out our books about the English language.

Subscribe to the Blog by email

Enter your email address to subscribe to the Blog by email. If you are an old subscriber and not getting posts, please subscribe again.

Categories
English English language Etymology Expression Grammar Language Linguistics Usage Word origin Writing

When Dickens don’t use ‘doesn’t’

[Note: An updated post on “don’t” and “doesn’t” appeared in June 2023.]

Q: While reading Dickens, I’ve noticed the use of “don’t” where we would now use “doesn’t.” In The Mystery of Edwin Drood, for example, the boastful auctioneer Thomas Sapsea says, “it don’t do to boast of what you are.”

A: What standard dictionaries say today about these contractions is fairly clear cut:

  • “Doesn’t” (for “does not”) should be used in the third person singular—with “he,” “she,” “it,” and singular nouns.
  • “Don’t” (for “do not”) is correct in all other uses—with “I,” “we,” “you,” “they,” and plural nouns. In the third person singular, “don’t” is considered nonstandard.

As you’ve noticed, however, it’s not unusual to find “don’t” used in place of “doesn’t” in 18th- and 19th-century fiction, like the example you found in that unfinished 1870 novel.

Was the usage ever “correct”? As is often the case with English, this is not a “yes or no” question.

In our opinion, this way of using “don’t” was always somewhat irregular (the Oxford English Dictionary suggests that it was regional or nonstandard from the start).

And as we’ll explain later, we think that in your example Dickens used “it don’t” colloquially to show that Mr. Sapsea didn’t speak the very best English.

The history of these contractions begins two centuries before Dickens. Both were formed in the 17th century, at a time when all forms of “do” were unsettled, to say the least.

For one thing, “does” and “doth”—both spelled in a variety of ways—were competing for prominence, as Merriam-Webster’s Dictionary of English Usage points out.

For another, some writers used the bare (or uninflected) “do” as the third person singular, according to M-W. The usage guide cites Samuel Pepys, writing in 1664: “the Duke of York do give himself up to business,” and “it seems he [the king] do not.”

With the verb itself so unsettled, it’s not surprising that the state of the contractions was even more chaotic.

In fact, M-W suggests that the use of the uninflected “do” for “does,” as in the Pepys citations, may have influenced the use of “don’t” as a contracted “does not.”

It’s significant that “don’t” was on the scene first; for a long while it was the only present-tense contraction for “do.” It was used as short for “do not” and (rightly or wrongly) for “does not.”

The earliest known written uses of “don’t” are from plays of the 1630s, though spoken forms were surely around long before that. And in the earliest OED examples, it’s used in the standard way—as short for “do not.”

The dictionary’s first example is dated 1633: “False Eccho, don’t blaspheme that glorious sexe.” (From Jasper Fisher’s Fuimus Troes, a verse drama; though published in 1633, it was probably performed a decade or so earlier.)

The next example is from William Cartwright’s The Ordinary, believed written about 1635: “Don’t you see December in her face?”

The OED also has a citation (with “I don’t”) from a comedy first acted in 1635 and published in 1640, Richard Brome’s The Sparagus Garden. And we’ve found a couple of interrogative uses (“dont you” and “dont they”) in a 1639 comedy, Jasper Mayne’s The City Match.

But “doesn’t,” with various spellings, wasn’t recorded until decades later—spelled “dozn’t” in 1678 and “doesn’t” in 1694, according to OED citations.

Even after “doesn’t” came on the scene, it apparently wasn’t common until at least a century later. Most uses of “doesn’t” that we’ve found in historical databases are from the 1760s or later, and it didn’t start appearing regularly (at least in writing) until the 1800s.

Before then, most writers used the uncontracted form, “does not,” even in fictional dialogue. The use of “don’t” in the third person singular was apparently irregular. The OED cites “he don’t,” “she don’t,” and “it don’t” among examples of regional or nonstandard uses, dating from 1660.

But to be fair, it seems only natural that mid-17th century British writers seeking a contraction for “does not” would use “don’t” in colloquial dialogue if “doesn’t” was unknown to them.

And no one can argue the fact that the earliest contraction people used for “does not” was “don’t.” Many continued to do so long after “doesn’t” came into the language.

M-W says, for example, that from the 17th through 19th centuries, the third person singular “don’t seems to have had unimpeachable status.” It cites examples (mostly in letters) by Horace Walpole, Charles Lamb, George Bernard Shaw, and Oliver Wendell Holmes.

Only after the usage was condemned in the latter half of the 19th century, M-W says, was this sense of “don’t” considered nonstandard.

We don’t agree entirely with M-W here. We’ve found hints that this use of “don’t” was regarded as less than exemplary by novelists of the 18th century.

For example, there are no irregular uses of “don’t” in Daniel Defoe’s Robinson Crusoe (1719), in his Moll Flanders (1722), or in Laurence Sterne’s Tristram Shandy (completed in 1767).

All three novels freely use “don’t” in the standard way and “does not” in the third person singular.

In Samuel Richardson’s novel Pamela (1740), we counted 14 examples of “don’t” in the third person singular—all but four used by servants—compared with 54 of “does not.”

We found no irregular uses of “don’t” in Henry Fielding’s Joseph Andrews (1742) and only two in his Tom Jones (1749)—spoken by a clerk and a servant.

Tobias Smollett’s Humphry Clinker (1771) has four uses of this irregular “don’t,” three by servants and one by an eccentric duke. Otherwise Smollett uses “does not” in the third person singular.

So apparently the principal novelists of the 18th century did not consider the third person singular “don’t” a normal usage, except sometimes among the rural or working classes. (None of them ever used “doesn’t” in writing, as far as we can tell.)

Even in 19th-century fiction, it’s mostly working-class characters who use “don’t” in a nonstandard way (though the occasional aristocrat uses it in a slangy, casual manner).

Let’s consider your quotation from Charles Dickens. When he wrote The Mystery of Edwin Drood, he deliberately put the nonstandard “it don’t” into the mouth Mr. Sapsea, a conceited fool who is convinced he’s brilliant and has pretensions to good breeding. The character is introduced with these words:

“Accepting the Jackass as the type of self-sufficient stupidity and conceit—a custom, perhaps, like some few other customs, more conventional than fair—then the purest Jackass in Cloisterham is Mr. Thomas Sapsea, Auctioneer.”

Sapsea isn’t the only character in the novel to use this irregular “don’t,” but the others are mostly laborers or servants. Those with higher education (teachers, clergy, etc.) use “does not.”

You don’t have to read 18th- or 19th-century fiction, however, to find nonstandard uses of “don’t.” They can be found in modern writing, too, mostly when the author intends to convey dialectal, regional, or uneducated English.

Graham Greene’s novel Brighton Rock (1938), for instance, has many examples in the speech of working-class characters: “That don’t signify” … “it don’t make any odds” … “it don’t seem quite fair.”

But modern British authors sometimes use this irregular “don’t” in portraying sophisticated, affluent characters who are deliberately (even affectedly) careless or casual in their speech.

Take, for example, Lord Peter Wimsey, the aristocratic, Oxford-educated detective in Dorothy L. Sayers’s novels of the ’20s and ’30s. He not only drops a “g” here and there (“an entertainin’ little problem”), but he often uses “don’t” in the third-person singular.

To cite just a handful of examples: “gets on your nerves, don’t it?” … “it don’t do to say so” … “when he don’t know what else to say, he’s rude” … “it don’t do to wear it [a monocle] permanently” … “it don’t do to build too much on doctors’ evidence” … “it don’t account for the facts in hand.”

Lord Peter isn’t an 18th-century character. He’s a 20th-century snob, and when he uses such English, he’s slumming linguistically.

Help support the Grammarphobia Blog with your donation.
And check out our books about the English language.

Subscribe to the Blog by email

Enter your email address to subscribe to the Blog by email. If you are an old subscriber and not getting posts, please subscribe again.

Categories
English English language Etymology Expression Language Phrase origin Politics Usage Word origin Writing

Birth of the conspiracy theory

Q: I read with interest your posts about “false flag” and “crisis actor.” But you used the term “conspiracy theory” without explaining its origin. I’ve read online that it was invented by the CIA after the assassination of JFK to discredit people who thought the shooter didn’t act alone.

A: The CIA did not invent the phrase “conspiracy theory.” It’s been in circulation since at least as far back as 1868, almost 100 years before President Kennedy was assassinated and nearly 80 years before the CIA existed.

Of course there have been conspiracy theories since ancient times—alternate views of history that interpret events as the products of secret conspiracies designed to conceal the truth.

One of the best known is the hypothesis that the Emperor Nero, for one reason or another, secretly orchestrated the Great Fire of Rome in 64 AD. (Modern scholars think the fire probably started by accident.)

But while conspiracies (both real and imagined) have always been a part of human history, it wasn’t until the 19th century that the phrase “conspiracy theory” was recorded in writing.

Before we get to the early examples of the expression, though, let’s look at its definition.

A “conspiracy theory,” according to the Oxford English Dictionary, is “the theory that an event or phenomenon occurs as a result of a conspiracy between interested parties.”

More specifically, the dictionary adds, it’s “a belief that some covert but influential agency (typically political in motivation and oppressive in intent) is responsible for an unexplained event.”

The definitions in standard dictionaries are similar though shorter, like this one: “A theory that explains an event or set of circumstances as the result of a secret plot by usually powerful conspirators” (Merriam-Webster Unabridged).

It’s interesting that the two earliest examples of “conspiracy theory” that we’ve found are from the same year but in different countries—the US and England.

The first is from a news story in the Boston Post on April 16, 1868:

“The testimony of Gen. Sherman has blown the conspiracy theory of Gen. Butler to the winds; and, of course, it was in a sure anticipation of such a result that he so steadily and brazenly objected to nearly every question put by the counsel for the defence which was calculated to bring it out.” (The testimony was given in the Senate’s impeachment trial of President Andrew Johnson.)

Later that same year, a British periodical printed the phrase in an article about a visit by Prime Minister Benjamin Disraeli to Queen Victoria at Balmoral Castle in Scotland:

“She may seem to award to her present Premier a degree of favour which, considering how direct and plain her dealings have ever been, appears to denote her sympathy with his policy, but she surely comprehends that his conspiracy theory is a mere party battle-horse for which she need not find stable room.” (The London Review of Politics, Society, Literature, Art, and Science, December 1868.)

In the 1870s and afterward, examples of “conspiracy theory” become much more common.

In April 1870, for example, another British periodical, The Journal of Mental Science, used the term in replying to allegations that mental patients were being severely beaten by keepers in insane asylums. The journal advanced another hypothesis to account for the patients’ injuries, and called the allegations of beatings a “conspiracy theory.”

Many sightings of “conspiracy theory” in the 1870s, ’80s, and ’90s are from news stories about crimes and courtroom proceedings. In such articles, it usually meant a hypothesis that an act was committed by more than one person.

This quotation, for example, is from a San Francisco newspaper’s account of a murder trial in which charges against a family of four were dismissed:

“The conspiracy theory was too intricate. He [the judge] was certain Blanche was not in it and how she could be left out he could not understand.” (The Daily Alta California, Aug. 30, 1873.)

And in a report about a far more sensational case, Henry Ward Beecher’s trial for adultery, this headline appeared: “How Bessie Turner’s Testimony Upsets the Conspiracy Theory.” (The Nashville Union and American, June 25, 1875.)

The first use of “conspiracy theory” in reference to a presidential assassination was in connection with the shooting of James A. Garfield by Charles Guiteau on July 2, 1881. (Garfield died several weeks later.)

This small headline appeared within one news report: “President Garfield and all His Cabinet Reject Conspiracy Theory.” The fact that New York police detectives had been called to Washington, the newspaper said, “started the sensational report that there had been a conspiracy to murder the President.” (Indianapolis Evening Star, July 4, 1881.)

The hypothesis—soon disproved—that the assassin did not act alone was also labeled “The Conspiracy Theory” in a headline on a different story published that same day in the Indianapolis Star.

Predictably, the expression got a good workout 20 years later after another president was assassinated.

By Sept. 8, 1901, two days after President William McKinley was shot by the anarchist Leon Czolgosz, “conspiracy theory” began popping up in the news. The theory here—that the shooting was an anarchist plot—was never proven. But it had some credibility, since Czolgosz admitted that he had been inspired by the writings of other anarchists.

For example, the headline “Conspiracy Theory Confirmed” appeared above a report that “an Italian” had been standing in front of Czolgosz until just before he fired the shots. (From a bulletin wired from London Sept. 8 and published the next day in the Adelaide Register in Australia.)

Citations in the OED haven’t yet caught up to these earlier sightings of “conspiracy theory.” The dictionary’s first example is from 1909:

“The claim that Atchison was the originator of the repeal may be termed a recrudescence of the conspiracy theory first asserted by Colonel John A. Parker of Virginia in 1880.” (From a review of a book, The Repeal of the Missouri Compromise, in the American Historical Review, July 1909.)

Published appearances of “conspiracy theory” continued right through the 20th century—with a spurt of activity after the JFK assassination—and on into our own time.

Along the way, a parallel term developed, “conspiracy theorist,” a noun phase that’s included in the OED and in standard dictionaries. Most don’t define it, however. An exception is the Cambridge Dictionary online: “someone who believes in a conspiracy theory.”

We haven’t found any examples of “conspiracy theorist” that predate the first citation given in the OED. It’s from the May 1, 1964, issue of the New Statesman:

“Conspiracy theorists will be disappointed by the absence of a dogmatic introduction.” (The comment was about a literary magazine’s transition to a broader coverage of the arts.)

Over the years, these terms have taken on a darker meaning. Today the “conspiracy” goes beyond the notion of someone’s acting with accomplices instead of alone. It also implies the involvement of entire governments or vast interests, not mere individuals.

Many of the OED’s citations reflect this broader use of “conspiracy theory,” like this one from the early 1950s:

“I call it the ‘conspiracy theory of society.’ It is the view that an explanation of a social phenomenon consists in the discovery of the men or groups who are interested in the occurrence of this phenomenon.” (From Karl Popper’s The Open Society and Its Enemies, rev. 2nd ed., 1952.)

And we found this early example from the 1970s of the wider meaning of “conspiracy theorist”:

“An invisible ‘supergovernment’ consisting of ‘organized crime, intelligence fronts, and war industry’ controls America, conspiracy theorist Rusty Rhodes told an audience of 250 last night in Cubberley Auditorium.” (The Stanford Daily, May 16, 1974.)

Rhodes, according to the article, went on to say that this “supergovernment … committed such wildly diverse acts as the assassination of President John Kennedy and the kidnaping [sic] of Patty Hearst.”

In short, the CIA did not invent the phrase “conspiracy theory.” And we’ve found no evidence that the agency tried to popularize it to make critics of the Warren Commission report look foolish.

Help support the Grammarphobia Blog with your donation.
And check out our books about the English language.

Subscribe to the Blog by email

Enter your email address to subscribe to the Blog by email. If you are an old subscriber and not getting posts, please subscribe again.

Categories
English English language Etymology Expression Language Punctuation Usage Word origin Writing

The cultured life of kefir

Q: I have heard the word “kefir” pronounced a number of ways. I would prefer to use a pronunciation that gives honor to its etymological origin. My research shows Caucasian languages as the possible source for the name. Any data on this topic that you would like to pass along?

A: “Kefir” is the English name for the fermented, yogurt-like drink made from cow’s milk, and its usual pronunciation in standard English dictionaries is keh-FEER.

Although etymologists say the term originated in the Caucasus, English speakers wouldn’t understand if you used a Caucasian pronunciation for the drink.

For example, the Georgian word for the drink, კეფირი, is kʼepiri in Latin script, while the Mingrelian word, ქიფური, is kipuri, according to the multilingual dictionary Glosbe.

In fact, English adopted the word from Russian, where the term for the drink, кефир, sounds much like the standard English pronunciation of “kefir.” The Russian term may ultimately come from a Turkic language spoken in the Caucasus.

The American Heritage Dictionary of the English Language (5th ed.) says the Russian term is derived “probably ultimately from Old Turkic köpür, (milk) froth, foam, from köpürmäk, to froth, foam.

Getting back to your question, we’d recommend using the standard English pronunciation, keh-FEER. If you were to walk into a grocery and ask for köpür, kʼepiri, or kipuri, the clerk wouldn’t know what you were talking about.

The Oxford English Dictionary, an etymological dictionary, defines “kefir” as an “effervescent liquor resembling koumiss, prepared from milk which has been fermented.” Koumiss is a drink made from fermented mare’s milk.

The earliest example of the term in the OED is from the July 3, 1884, issue of Nature: “Kephir has only been generally known even in Russia for about two years.”

The next citation is from the Nov. 3, 1894, issue of the Lancet: “Koumiss and kefyr are examples of sour fermented milk containing an excess of carbonic acid gas.” (After checking the original, we corrected an OED typo—“are examples,” not “and examples.”)

Help support the Grammarphobia Blog with your donation.
And check out our books about the English language.

Subscribe to the Blog by email

Enter your email address to subscribe to the Blog by email. If you are an old subscriber and not getting posts, please subscribe again.

Categories
English English language Expression Grammar Language Linguistics Usage Writing

How to say you’re not quite sure

Q: What is the difference in meaning between “John didn’t come yesterday—he must have been ill” and “John didn’t come yesterday—he will have been ill”? I realize that “must” is more popular than “will” in such constructions, but does one express more certainty than the other?

A: The words “will” and “must” in your examples are epistemic modal verbs, auxiliary verbs that express probability.

As Rodney Huddleston and Geoffrey K. Pullum explain in the Cambridge Grammar of the English Language, “epistemic modality qualifies the speaker’s commitment to the truth.”

“While It was a mistake represents an unqualified assertion,” Huddleston and Pullum write, “It must have been a mistake suggests that I am drawing a conclusion from evidence rather than asserting something of whose truth I have direct knowledge.”

In your first example (“John didn’t come yesterday—he must have been ill”), the auxiliary “must” indicates that the writer (or speaker) believes John was probably ill.

In our opinion, the expression “he will have been ill” indicates somewhat more probability than “he must have been ill” (though some might argue the point). And both of them indicate a much greater probability than “he may have been ill”—another example of epistemic modality.

Huddleston and Pullum note that epistemic modality is “commonly expressed by other means than modal auxiliaries.” For example, by adverbs (“he was probably ill”), verbs (“I believe he was ill”), adjectives (“he was likely to be ill”), and nouns (“in all likelihood, he was ill”).

There are two other principal kinds of modality: deontic, which expresses permission or obligation (“He may have one more chance, but he must come tomorrow”), and dynamic, which expresses willingness or ability (“I won’t come today, but I can come tomorrow”).

In A Comprehensive Grammar of the English Language, the authors, Randolph Quirk et al., say, “At its most general, modality may be defined as the manner in which the meaning of a clause is qualified so as to reflect the speaker’s judgment of the likelihood of the preposition it expresses being true.”

Quirk divides the modal verbs into two types:

“(a) Those such as ‘permission,’ ‘obligation,’ and ‘volition’ which involve some kind of intrinsic human control over events, and

“(b) Those such as ‘possibility,’ ‘necessity,’ and ‘prediction,’ which do not primarily involve human control of events, but do typically involve human judgment of what is or is not likely to happen.”

Quirk adds that the two categories “may be termed intrinsic and extrinsic modality respectively,” since “each one of them has both intrinsic and extrinsic uses: for example, may has the meaning of permission (intrinsic) and the meaning of possibility (extrinsic); will has the meaning of volition (intrinsic) and the meaning of prediction (extrinsic).”

“However, there are areas of overlap and neutrality between the intrinsic and extrinsic senses of a modal: the will in a sentence such as I’ll see you tomorrow then can be said to combine the meanings of volition and prediction.”

Another point to consider, Quirk writes, “is that the modals themselves tend to have overlapping meanings, such that in some circumstances (but not in others), they can be more or less interchangeable.”

In other words, there’s a lot of ambiguity here. Or, as Quirk puts it, “the use of modal verbs is one of the more problematic areas of English grammar.”

Help support the Grammarphobia Blog with your donation.
And check out our books about the English language.

Subscribe to the Blog by email

Enter your email address to subscribe to the Blog by email. If you are an old subscriber and not getting posts, please subscribe again.

Categories
English English language Etymology Expression Language Phrase origin Usage Word origin Writing

Are your ears burning?

Q: I wonder if “Are your ears burning?” is an expression that you may want to parse.

A: The expression is derived from an old belief that one’s ears can somehow sense that one is being talked about, even if the talking is going on at a distance.

The ears supposedly respond to such gossip by burning or glowing or ringing or some other physical change.

The Oxford Dictionary of English Idioms (3d ed., 2009), edited by John Ayto, says “someone’s ears are burning” means “someone is subconsciously aware of being talked about, especially in their absence.”

But The American Heritage Dictionary of Idioms (1997), written by Christine Ammer, defines “one’s ears are burning”  as being troubled by overhearing an actual conversation:

“Be disconcerted by what one hears, especially when one is being talked about.”

As far as we can tell, the belief that one’s ears can sense something said about one in absentia first showed up in the writings of the first-century Roman philosopher Pliny the Elder.

In Naturalis Historia (also known as Historia Naturalis), Pliny’s 37-volume encyclopedia of fact, myth, and speculation, he writes:

Quin et absentes tinnitu aurium præsentire sermones de se receptum est (“Those absent are warned by a ringing of the ears when they are being talked about,” from volume 28, chapter 5).

The first example of the usage we’ve seen in English is from Troilus and Criseyde, a Middle English poem written by Chaucer in the 1380s:

“And we shal speek of the somwhat, I trowe, / Whan thow art gon, to don thyn eris glowe” (“And when thou art gone, I trust, we shall speak of thee somewhat to make thine ears glow”).

The earliest example we’ve come across that specifically mentions burning is from Of the Burning of the Eares, a 16th-century poem by James Yates:

“That I doe credite give unto the saying old: / Which is, when as the eares doe burne, some thing on thee is told” (from The Castell of Courtesie, a 1582 collection of Yates’s poetry).

The first example we’ve seen for the version of the expression you cite (“Are your ears burning?”) is from the July 6, 1892, issue of Our Church Paper, a Lutheran weekly in New Market, VA.

The paper reprinted a letter from Japan to children in the states. The letter, apparently written by the father of the children, suggests that their ears may be burning because he’s been thinking of them:

“Well, children, are your ears burning today? Whether they are or not, I have been thinking about you a great deal. For I have read over again the letters that came from childish pens away across the sea, in order that I might answer some of the questions you have asked me.”

There have been many related superstitions, such as that the ringing of the right ear signifies you’re being praised, while the ringing of the left indicates you’re being criticized. Enough said. We’re up to our ears.

Let’s end with an example from Shakespeare of an actual conversation that’s overheard. In Much Ado About Nothing, believed written in the late 1590s, Beatrice’s ears burn when she overhears Hero and Ursula speaking of her:

What fire is in mine ears? Can this be true?
Stand I condemned for pride and scorn so much?
Contempt, farewell! and maiden pride, adieu!
No glory lives behind the back of such.

Help support the Grammarphobia Blog with your donation.
And check out our books about the English language.

Subscribe to the Blog by email

Enter your email address to subscribe to the Blog by email. If you are an old subscriber and not getting posts, please subscribe again.

Categories
English English language Etymology Expression Language Phrase origin Usage Word origin Writing

Roots of the grapevine

Q: I assume the word “grapevine” originally referred to a vine on which grapes grow. When did it come to mean a casual way of passing information along, as in the Motown song of the ’60s?

A: Yes, “grapevine” did originally refer to Vitis vinifera, the vine of the common grape. It first showed up in New England in the mid-17th century as two words.

The Oxford English Dictionary’s earliest citation, which we’ll expand here, is from a Jan. 19, 1654, entry in the town records of Providence, RI:

“five Acres of Low Land Layd out on the South side of the West River for Robert Pike to make Medow, bounded on the West End with a black Oak markt on 4. Sides, and on the East end on the lower side (that small slipe of low Land) by the grape Vines.”

The use of “grapevine” to mean an informal source of information began life in the mid-19th century as “grapevine telegraph,” according to the Random House Historical Dictionary of American Slang.

The slang dictionary defines the usage as “any informal or unofficial method of relaying important or interesting information, esp. by word of mouth,” or “the means by which gossip or rumor travels.”

When “grapevine telegraph” first appeared in the early 1850s, it referred to the transmission of questionable or outdated information.

The earliest examples we’ve found in searches of digitized newspaper databases are from two weeklies that were published on the same date—April 17, 1852—in different states.

This one is from the editorial page of the Freeman (Fremont, OH): “The following important dispatch was recived [sic] by the ‘Grape Vine’ telegraph—It came to hand just in time for this week’s ‘issoo,’ and the attentive ‘operator’ at Tiffin city has our thanks for his invaluable favor.”

What follows is an anonymous, satirical letter to the editor, full of outlandish misinformation about an election for township offices.

The second example published that day is from the Wabash Courier (Terre Haute, IN): “the aforesaid writer is awfully behind the times, a resident, perhaps, of some deep diggings where the sun never shines, and the inhabitants get their news by the grapevine telegraph. It would be no easy matter to ascertain the number of people badly fooled.”

The fact that the expression appeared on the same day in different states suggests that it was around much earlier in speech.

It shows up again a couple of months later, on June 30, 1852, in another Terre Haute weekly, the Wabash Express: “We suppose the information came from Maine and California (the Aroostook and Sierra Nevada,) by the mud turtle line, or the grapevine telegraph.”

And the July 17, 1852, issue of the Wabash Courier uses the phrase to introduce a mock letter from James Buchanan, then a former secretary of state and later a president, to Senator Lewis Cass of Michigan:

From the Pittsburgh Despatch. By the Grape Vine Telegraph line, in connection with Virginia Fence and Mason & Dixon’s Line, we have received the following interesting correspondence—far ‘ahead of the foremost,’ which we hasten to lay before our readers.”

(The word sleuth Barry Popik has found an earlier reprint of that passage in a New Jersey newspaper, the Trenton State Gazette, June 22, 1852. We haven’t been able to find the original Pittsburg Dispatch article.)

During the American Civil War, “grapevine telegraph” was used to describe doubtful information about the war that was passed along from mouth to mouth. Union supporters often used the phrase for what they considered Confederate propaganda, as in this example from the Jan. 8, 1862, Evansville (IN) Daily Journal:

“Some of those who are so very anxious that the South should have its rights, were very much elated at the news by grapevine Telegraph from Evansville via Phillipstown, to the effect that there had been a fight in Kentucky, in which the Federals were badly whipped, and were rushing into Evansville by the thousands, perfectly panic-struck.”

And this example comes from the June 12, 1862, Daily Alta California (San Francisco): “Yesterday the rebel grapevine telegraph was actively employed. It gave out oracularly that [Maj. Gen.] Sterling Price had crossed the Tennessee River near Florence, Ala., with 10,000 men, and was making his way to Nashville.”

The OED notes that the word “grapevine” was also used by itself during the war to mean a false or unfounded rumor or story. The dictionary cites a passage, which we’ve expanded, from “The Old Sergeant,” an 1863 poem by Forceythe Willson.

In the poem, a dying Union soldier imagines that he wasn’t really wounded at Shiloh and that his memory of being cut down is all in his mind: “It’s all a nightmare, all a humbug and a bore; / Just another foolish grape-vine—and it won’t come any more.”

Random House has a somewhat earlier example that uses both long and short versions of the expression:

“We get such ‘news’ in the army by what we call ‘grape vine,’ that is, ‘grape vine telegraph.’ It is not at all reliable.” (From an 1862 diary entry by James A. Connolly, first published in the Transactions of the Illinois State Historical Society, 1928. Connolly enlisted as a private in the Union Army and rose to the rank of lieutenant colonel.”

The next Oxford example is from a postwar article by Horace Carpenter, a former lieutenant in the Ninth Louisiana Battalion, about life at Johnson’s Island, a prison-of-war camp for Confederate officers in Lake Erie near Sandusky, OH. Here Carpenter discusses rumors of prisoner exchanges:

“The ‘grape-vine’ spoke to us of little else. The main feature of this prison telegraph was its complete unreliability. As I remember, it was never correct, even by accident.” (We’ve expanded the citation, from the March 1891 issue of the Century Illustrated Monthly Magazine.)

Today the word “grapevine” refers to the informal transmission of information that may or may not be true. As the OED explains, “Now in general use to indicate the route by which a rumour or a piece of information (often of a secret or private nature) is passed.”

The earliest Oxford example for this sense is from The Young Manhood of Studs Lonigan (1934), the second novel in James T. Farrell’s Studs Lonigan trilogy: “Down there at that express company they find out about everything a guy does. They got the best grapevine in the world.

Random House has an earlier example from John Brown’s Body, a 1928 poem by Stephen Vincent Benét: “And the grapevine whispered its message faster / Than a horse could gallop across a grave.”

In a 2009 post, we dismissed the notion that the usage ultimately comes from The Old Grapevine, a Greenwich Village tavern frequented by politicians, artists, and intellectuals.

We’ve found no evidence that the tavern was the source of the old usage, though it may have helped popularize it. We also haven’t seen evidence that the usage was inspired by hastily strung early telegraph wires that twisted like grapevines.

The expression “grapevine telegraph” showed up in the US as telegraph wires were spreading across the country. Samuel Morse sent his famous message, “What hath God wrought,” from Washington to Baltimore in 1844.

We suspect that the simple presence of telegraph wires inspired the figurative use of “grapevine telegraph” for the informal transmission of information. As Merriam-Webster Unabridged explains, the usage was “probably so called from the grapevine’s being thought of as a humble substitute for a telegraph line.”

A similar expression, “clothesline telegraph,” appeared during the Civil War. The March 22, 1862, Cambridge (MA) Chronicle describes Southern women who spread Confederate propaganda as “tattlers, and operators upon the clothesline telegraph, mischief makers.”

Similarly, the figurative expression “bush telegraph” showed up in the late 19th and “jungle telegraph” in the early 20th century.

In his 1901 autobiography Up From Slavery, Booker T. Washington writes that slaves on the plantation where he grew up used the “grapevine” expression for their whispered conversations about the war:

“Though I was a mere child during the preparation for the Civil War and during the war itself, I now recall the many late-at-night whispered discussions that I heard my mother and the other slaves on the plantation indulge in. These discussions showed that they understood the situation, and that they kept themselves informed of events by what was termed the ‘grape-vine’ telegraph.”

However, we haven’t seen any written wartime examples for “grapevine” or “grapevine telegraph” used to describe such slave discussions. And as we’ve said, “grapevine telegraph” was often used during the war in the sense of Confederate propaganda.

We’ll end with a few lines from Marvin Gaye’s version of “I Heard It Through the Grapevine,” the Motown song, written by Norman Whitfield and Barrett Strong, that was a hit in the 1960s for Gaye as well as Gladys Knight & the Pips:

You could have told me yourself
That you loved someone else
Instead I heard it through the grapevine
Not much longer would you be mine
Oh, I heard it through the grapevine
And I’m just about to lose my mind

Help support the Grammarphobia Blog with your donation.
And check out our books about the English language.

Subscribe to the Blog by email

Enter your email address to subscribe to the Blog by email. If you are an old subscriber and not getting posts, please subscribe again.

Categories
English English language Grammar Language Usage Writing

When the simple past is perfect

Q: I don’t see why the past perfect, not the present perfect, is used in the following sentence: “The Putinversteher, who until recently had dominated the German media and are still heavily present in the German-language book market, have contributed to this development.”

A: We wouldn’t use either the present perfect (“have dominated”) or the past perfect (“had dominated”). We’d use the simple past tense (“dominated”):

“The Putinversteher, who until recently dominated the German media and are still heavily present in the German-language book market, have contributed to this development.”

The present perfect (e.g., “have dominated”) generally refers to an action that began in the past and continues into the present, or to one that occurred at some indefinite time in the past. The past perfect (“had dominated”) is used to differentiate a past action from one that happened at a more recent time in the past.

Neither perfect tense applies in this case.

The key passage here refers to one specific past action, which the author describes as the recent domination of the German media by the Putinversteher (German for “those who understand Putin”). The use of “until recently” shows that the action ended before the current time, so the simple past tense is appropriate.

The passage, as you undoubtedly know, is from a Jan. 21, 2016, article in the Harvard International Review by the German political scientist Andreas Umland. The article, translated from German, criticizes the German defenders of the Russian leader Vladimir Putin.

The article could have used the present perfect if the domination continued into the present (“… who have dominated the German media and are heavily present in the German-language book market …”) or if the domination was for an indefinite time in the past (“… who have sometimes dominated the German media and are heavily present in the German-language book market …”).

The past perfect could have been used if one past action preceded another (“… who had dominated the German media and were still heavily present in the German-language book market …”).

Help support the Grammarphobia Blog with your donation.
And check out our books about the English language.

Subscribe to the Blog by email

Enter your email address to subscribe to the Blog by email. If you are an old subscriber and not getting posts, please subscribe again.

Categories
English English language Etymology Expression Language Phrase origin Politics Usage Word origin Writing

Backstage with ‘crisis actors’

Q: Thanks for answering my question about “false flag.” Now, who came up with the term “crisis actor”? I can’t believe what’s happening to the English language. This whole country has gone crazy. It makes you want to pull out your brain and give it a good shake.

A: Now that you’ve vented, let’s take a look at “crisis actor,” a relatively new term that’s being used to question the legitimacy of victims or survivors of mass shootings.

When the phrase first appeared in print 50 years ago, “crisis actor” was a term in political science that referred to a country in conflict.

The earliest example we’ve seen is from the Proceedings of the International Peace Research Association Second Conference: Poverty, Development and Peace (1968):

“McClelland argues that performance characteristics of crisis actors, both in crisis and non-crisis behavior, can be identified, and that phase characteristics of particular crises and of crises in general as one type of international behavior, can be distinguished.”

The reference is to a 1965 paper (“Systems Theory and Human Conflict”) by the political scientist Charles A. McClelland. Though he refers to countries in conflict as “actors” in “crisis,” he doesn’t actually use the phrase “crisis actor” or “crisis actors.”

The phrase took on a new sense a few months after the July 20, 2012, mass shooting inside a movie theater in Aurora, CO.

On Oct. 31, 2012, Visionbox, a nonprofit acting group in Denver, issued a press release announcing that it was offering actors to help shopping malls prepare for dealing with the victims of mass shootings:

“Visionbox Crisis Actors are trained in criminal and victim behavior, and bring intense realism to simulated mass casualty incidents in public places,” the press release says.

The release adds that the actors can “help first responders visualize life-saving procedures, and assist trainers in delivering superior hands-on crisis response training.”

However, the phrase was soon co-opted by people who questioned the official version of the Dec. 14, 2012, mass shooting at the Sandy Hook Elementary School in Newtown, CT.

The earliest written example we’ve seen for the new usage is in a Dec. 25, 2012, article on the Washington Examiner website that includes a “partial list of interesting questions being raised all over the internet” about Sandy Hook, including this one:

“Was the school part of the shooting spree an emergency response exercise using paid crisis actors funded by a grant from our federal government?” The article has a link to the Visionbox press release.

The now-deleted article, by Lori Stacey, was captured on the Internet Archive Wayback Machine and tracked down by the linguist Ben Zimmer.

“That tenuous chain of reasoning was enough for conspiracy theorists to begin imagining that Newtown was a staged event populated with ‘crisis actors,’ ” Zimmer wrote in a March 2, 2018, article in the Wall St. Journal.

Days before the Washington Examiner piece appeared, people were raising questions online about Sandy Hook, as Jason Koebler points out in a Feb. 22, 2018, article on the Motherboard website.

For example, James Tracy, a professor of communications at Florida Atlantic University, suggested in a Dec. 20, 2012, post on his Memory Hole Blog that the official Sandy Hook account was a “meticulously crafted façade” with a “made-for-television storyline.”

Tracy, who was later fired by the university, didn’t use the phrase “crisis actor” in that post but a Dec. 22 comment to it used the term “actors” in suggesting that the mass shooting was staged by broadcasting “gunshots and mayhem over the intercom system.”

The commenter, using the handle “andrew.w,” says such a broadcast “adds a bit of zing to what is essentially a drill and allows the actors and children to actually relate what they heard and did with a bit more reality.”

Enough. We’d better stop here or we’ll have to pull out our brains and give them a good shake.

Help support the Grammarphobia Blog with your donation.
And check out our books about the English language.

Subscribe to the Blog by email

Enter your email address to subscribe to the Blog by email. If you are an old subscriber and not getting posts, please subscribe again.

Categories
English English language Etymology Expression Language Phrase origin Usage Word origin Writing

Soul of discretion

Q: What is the origin of the phrase “soul of discretion”? Google just leads to its being an idiom, and I thought you might know how to dig more deeply.

A: When the word “soul” appeared in Anglo-Saxon times, it referred to the “essential principle or attribute of life,” according to the Oxford English Dictionary.

The OED has several early Old English examples, including this one from the Vespasian Psalter, an illuminated manuscript written in the 700s, with the Psalms in Latin and Old English:

“Conturbata sunt omnia ossa mea et anima mea turbata est ualde: gedroefed sindun all ban min & sawl min gedroefed is swiðe” (“All my bones are troubled, and my soul is troubled sorely,” Psalm 6:2).

In the late 1500s, the noun “soul” came to mean the personification of some admirable quality or thing.

The first Oxford example is from The First Part of Ieronimo, an anonymous play published in 1605 but probably written in the 1580s: “Prince Balthezer. … The very soule of true nobility.”

(The play, about a Spanish knight marshal, may have inspired, or been inspired by, the English playwright Thomas Kyd’s The Spanish Tragedy, believed written sometime in the 1580s.)

And here’s an example from Shakespeare’s Timon of Athens, possibly written sometime before 1610, perhaps with the playwright Thomas Middleton: “O he’s the very soule of Bounty.”

The OED also has an example of “soul” used in the exact sense you’re asking about, as the personification of discretion. It’s from You May Now Kill the Bride, a 2006 mystery by the American novelist Deborah Donnelly: “I haven’t always been the soul of discretion myself.”

However, the usage has been around for much longer. We found this example in Phyllis of Philistia, an 1895 romantic novel by the Irish writer Frank Frankfort Moore: “ ‘You are the soul of discretion, my beloved,’ said the husband.”

Help support the Grammarphobia Blog with your donation.
And check out our books about the English language.

Subscribe to the Blog by email

Enter your email address to subscribe to the Blog by email. If you are an old subscriber and not getting posts, please subscribe again.

Categories
English English language Etymology Expression Language Linguistics Usage Word origin Writing

The truth about trees

Q: The words “tree” and “true,” according to a TED video, have a common ancestor. From what I gather, this was back in prehistoric times, before there was writing. So how do we know the first thing about an ancient language if there’s no written record of it?

A: Historical linguists believe that “tree” and “true” have a common prehistoric ancestor, a belief that’s based on studies of a reconstructed, hypothetical ancient language known as Proto Indo-European (PIE for short), not on any written evidence.

By studying members of the present Indo-European family, linguists have extrapolated back to the presumed prehistoric language. The Indo-European family comprises many European and Asian languages, including English.

In reconstructing the PIE vocabulary, for example, linguists have used the comparative method, a technique for finding similar words in various Indo-European languages.

As the linguist and phonologist Calvert Watkins explains, similar words, or cognates, in present Indo-European languages “provide evidence for the shape of the prehistoric Indo-European word.”

Watkins, author of The American Heritage Dictionary of Indo-European Roots, says “tree” and “true” are ultimately derived from deru-, a Proto Indo-European root meaning to “be firm, solid, steadfast.”

This PIE root gave prehistoric Germanic the terms trewam (“tree”) and treuwithō (“truth”), and Old English the words trēow (“tree”) and trēowe (“true”), Watkins writes. (Old English spellings vary considerably.)

The earliest example of “tree” in the Oxford English Dictionary (with the plural treo) is from the Vespasian Psalter, an illuminated manuscript dated around 825:

“Muntas and alle hyllas, treo westemberu and alle cederbeamas” (“Mountains and all hills, fruit trees and all cedars”). We’ve expanded the citation, from Psalm 148.

And in Pastoral Care, King Alfred’s late ninth-century translation of a sixth-century work by Pope Gregory, an unbeliever is compared to a barren tree:

“Ælc triow man sceal ceorfan, þe gode wæstmas ne birð, & weorpan on fyr, & forbærnan” (“Every tree that does not bear good fruit shall be cut down and cast into the fire and burnt”). We’ve expanded the OED citation, which refers to Matthew 7:19. The dictionary describes triow here as a variant reading of treow.

When “true” showed up in Old English, it meant loyal, faithful, or trustworthy. Here’s an example from Beowulf, an epic poem that may have been written as early as 725:

“Þa gyt wæs hiera sib ætgædere, æghwylc oðrum trywe” (“The two were at peace together, true to each other”). Here trywe is a variant spelling of trēow.

As Old English gave way to Middle English in the 12th century, the word “true” came to mean accurate or factual, as in this example from Layamon’s Brut, an early Middle English poem written sometime before 1200:

“Belin ihærde sugge þurh summe sæg treowe of his broðer wifðinge” (“Belin heard it said through some true report of his brother’s marriage”). The passage refers to Belin and Brennes, brothers who vie in Anglo-Saxon legend to rule Britain.

And this example is from Wohunge Ure Lauerd (“The Wooing of Our Lord”), a Middle English homily written sometime before 1250. The author, presumably a woman, tells Jesus of her passionate love for him:

“A swete ihesu þu oppnes me þin herte for to cnawe witerliche and in to reden trewe luue lettres” (“Ah, sweet Jesus, you open your heart to me, so that I may know it inwardly, and read inside it true love letters”).

Help support the Grammarphobia Blog with your donation.
And check out our books about the English language.

Subscribe to the Blog by email

Enter your email address to subscribe to the Blog by email. If you are an old subscriber and not getting posts, please subscribe again.

Categories
English English language Etymology Expression Language Phrase origin Usage Word origin Writing

What about ‘whatnot’?

Q: For as long as I can remember, I’ve heard the phrase “what not” used much like “et cetera.” I’m curious about the etymology. I searched your archives but can’t find that you’ve written about it. Have you?

A: No, we haven’t written about it yet, so let’s remedy that now.

You may be surprised to hear this, but the term “whatnot” (it’s usually one word today) has been around for hundreds of years, dating back to the mid-1500s.

When the usage first appeared, as two words, it could mean “anything,” “everything,” “anything and everything,” or “all sorts of things,” according to the Oxford English Dictionary.

The earliest OED example is from The Comedye of Acolastus, John Palsgrave’s 1540 translation of a Latin carnival play written in 1529 by the Dutch writer Gulielmus Gnapheus:

“Excesse of fleshely pleasures … hath taken awaye all thynges … my goodes or substance, my name .i. my good name and fame, my frendes, my glory .i. my renoume or estimation, what not? .i. what thyng is it that she hath not taken from me?” (Palsgrave uses the abbreviation “.i.” for the Latin id est, or “that is,” usually rendered as  “i.e.”)

Today, according to the dictionary, “whatnot” is used as a “final item of an enumeration” and means “anything else, various things besides; ‘whatever you like to call it.’ ”

The first Oxford example for the term used as a final item in a series is from a Dec. 21, 1663, entry in the diary of Samuel Pepys: “The strange variety of people … bakers, brewers, butchers, draymen, and what not.”

Since the early 1800s, the OED says, the term has also been used for an “article of furniture consisting of an open stand with shelves one above another, for keeping or displaying various objects, as ornaments, curiosities, books, papers, etc.”

The dictionary’s first example of the term used for an open stand for bric-a-brac is from a Dec. 21, 1808, letter by Lady Sarah Spencer (later Baroness Lyttelton) to her brother, Bob, a 16-year-old midshipman in the Royal Navy and later Capt. Sir Robert Cavendish Spencer. Note her italics for “what-nots,” suggesting the usage was relatively new:

“There is a new and very handsome thick carpet put down in the old library; of course therefore we breakfasted in the drawing-room, while all the old chairs, tables, what-nots, and sofas were torn up by the roots to make room for the new-comer.” (We’ve expanded the OED citation.)

Help support the Grammarphobia Blog with your donation.
And check out our books about the English language.

Subscribe to the Blog by email

Enter your email address to subscribe to the Blog by email. If you are an old subscriber and not getting posts, please subscribe again.

Categories
English English language Etymology Expression Language Linguistics Phrase origin Slang Usage Word origin Writing

Wolf tickets for sale

Q: I recently heard a television commentator use the phrase “selling wolf tickets.” After research, I found both a Russian and an African-American Vernacular English source for somewhat related phrases. Did these evolve independently or is there evidence for cross pollination?

A: To “sell wolf tickets,” an expression that’s about 60 years old, is to oversell yourself—to spread boasts or threats that you can’t (or won’t) back up.

The usage was first recorded in writing in 1963, when sociologists noted its use by Black gang members in Chicago. The sociologists had reported it two years before in a speech, and it was undoubtedly used on the streets even earlier than that.

Some commentators have suggested that the expression comes from “to cry wolf” (to bluff or raise a false alarm). But a more likely theory is that the “wolf” here was originally “woof” and was intended to mean a bark without a bite.

In African-American Vernacular English, to “woof” has meant to bluff or challenge since at least as far back as 1930. In fact, the phrase has been recorded as “sell woof tickets” since the 1970s.

But as we said, the earliest written example we’ve found for the complete phrase is the “wolf” version; this may reflect the way “woof” was interpreted by white sociologists in the mid-20th century.

Let’s start with “woof” and come later to “sell wolf [or woof] tickets.”

The Dictionary of American Regional English defines the verb “woof,” used “chiefly” among Black speakers, as “to engage in behavior, esp speech, intended to impress, intimidate, or provoke; to bluff, kid.” DARE also mentions “woofer,” “woofing,” and other related words.

The dictionary’s earliest example of “woof” used in this way comes from a play written in 1930: “Stop woofing and pick a little tune there so that I can show Daisy somethin’.” (From Mule Bone, written in Black vernacular, by Zora Neale Hurston and Langston Hughes.)

In December 1934, the journal American Speech published a paper mentioning both “woof” and “woofer” as terms in Black college slang. Here are the examples:

“WOOF. To talk much and loudly and yet say little of consequence,” and “A WOOFER. Applied to one who talks constantly, loudly, and in a convincing manner, but who says very little.” (From “Negro Slang in Lincoln University,” a paper by Hugh Sebastian.)

The earliest published example we’ve found for “sell wolf tickets” is from a 1963 paper on the sociology of gang behavior, though an unpublished version dates from 1961. Here’s the relevant passage (“worker” is a social worker and “Commando” a gang leader):

“In a conflict situation, without a worker present, Commando would find it difficult not to ‘sell wolf tickets’ (i.e., challenge) to rival gang members and instigate conflict.”

The paper, “The Response of Gang Leaders to Status Threats: An Observation on Group Process and Delinquent Behavior,” by James F. Short, Jr. and Fred L. Strodtbeck, was published in the American Journal of Sociology in March 1963. This was a revised version of a paper (now lost) read on Sept. 1, 1961, at the annual meeting of the American Sociological Association.

In answer to an email query, Dr. Short told us that the same passage, with the phrase “sell wolf tickets,” probably appeared in the earlier, unpublished version that was delivered in 1961. “I cannot imagine that it was not in the earlier version,” he said.

Over the years, both “wolf tickets” and “woof tickets” have appeared, with variant spellings for “woof” and with “tickets” in singular as well as plural.

DARE, for example, says the phrase “woof ticket,” used “especially” among Black speakers, means a “lie, bluff, challenge.” Its earliest written use was recorded in 1971.

The scholar Geneva Smitherman, writing in the English Journal in February 1976, wrote: “ ‘Sellin woof [wolf] tickets’ (sometimes just plain ‘woofin’) refers to the kind of strong language which is purely idle boasting.” The bracketed and parenthetical additions are hers.

Time magazine also used both versions in its Aug. 20, 1979, issue: “ ‘To sell wolf tickets’ (pronounced wuf tickets) means to challenge somebody to a fight” (from “Outcry Over Wuff Tickets,” an article about Black English in the classroom).

And in 1982, an early rap group called Wuf Ticket briefly appeared on the singles charts.

A few years later, the linguist Carolyn G. Karhu said that “wolf ticket” (defined as an empty threat) and “selling wolf tickets” (making an empty threat) were terms used by prison inmates in Tennessee (American Speech, summer 1988).

But by the 1990s, these terms had apparently become passé in the language of the streets.

“ ‘Woof ticket’ is a somewhat dated phrase,” Betty Parham and Gerrie Ferris wrote in 1992 in the Atlanta Journal and Constitution. And “selling wolf tickets” was defined as “archaic Black slang” by Jack E. White, writing in Time magazine’s issue of Oct. 24, 1995.

So was this “woof” merely a Black pronunciation of “wolf”? The language columnist William Safire thought so. Commenting on the phrase “woof ticket,” he wrote, “Woof is a Black English pronunciation of ‘wolf’ ” (the New York Times Magazine, Nov. 5, 2000).

That assertion brought a response from Peter Jeffery, now a professor of medieval studies at the University of Notre Dame. His letter, later published in Safire’s book The Right Word in the Right Place at the Right Time (2004), objected to Safire’s explanation for “woof ticket.”

“The origin had nothing to do with ‘wolf,’ ” Jeffery wrote. “The metaphor was of a barking watchdog (‘woof, woof!’).”

Jeffery, who grew up in Brooklyn and heard the phrase as a youth, added: “By the 1970s, ‘woof ticket’ had disappeared from the speech of young Black Americans, though it may still be remembered among those are old enough.” He noted, “I’ve since heard that ‘woofin’ is still sometimes used among jazz musicians to describe the back-and-forth challenges between instrumental soloists.”

Well, old slang terms have a way of reviving, and that appears to be the case here.

Today the phrase is usually seen as “to sell wolf tickets,” and its meaning has become broader. It’s sometimes used to describe a hyped-up promotion or an inflated sales pitch—for a product or event that doesn’t live up to the hype.

By the way, we’ve found no connection with the use of the phrase in Russian slang, where “wolf ticket” (волчий билет or volchiy bilyet) refers to a document or other impediment that negatively affects someone, such as by making it impossible to get a specific job or enroll in a certain university.

[Updated on Dec. 10, 2018]

Help support the Grammarphobia Blog with your donation.
And check out our books about the English language.

Subscribe to the Blog by email

Enter your email address to subscribe to the Blog by email. If you are an old subscriber and not getting posts, please subscribe again.

Categories
English English language Etymology Expression Language Phrase origin Politics Usage Word origin Writing

The true history of false flags

Q: I simply don’t understand how “false flag” has come to mean (I guess) a staged tragedy to create sympathy for a group or to push an agenda such as outlawing automatic weapons. It supposedly relates back to pirate ships flying false flags, but that doesn’t seem parallel.

A: Yes, the term “false flag” conjures up images of pirates on the high seas, flying friendly colors to conceal their larcenous motives. However, we haven’t found any evidence that the phrase was ever used literally for a real flag on a real pirate ship.

In the 16th century, when the phrase first appeared in writing, it was strictly a figurative expression. It wasn’t used literally—to mean an actual flag—until almost 300 years later. And pirates weren’t mentioned.

“False flag” is one of those expressions that exist almost solely in figurative use. And its meaning hasn’t changed over the centuries.

In figurative contexts, the Oxford English Dictionary says, a “false flag” means “a deliberate misrepresentation of someone’s affiliation or motives; something used deliberately to misrepresent in this way.”

It first appeared, according to the OED, in a religious tract published in the mid-1500s: “Of this sort was Gardiner that abused K. Henry with a false flagge of religion, when he made hys booke of true obedience” (from Thomas Norton’s A Warning Agaynst the Dangerous Practises of Papistes, 1569).

At that time, the word “flag” itself was still new. The noun for a square or rectangular piece of cloth, varying in color and design and flown “as a standard, ensign or signal, and also for decoration or display,” first appeared in writing in 1530, the OED says.

The dictionary’s next example of “false flag” is also figurative, though the writer alludes to pirates. In a sermon published in 1689, George Halley called Roman Catholicism “a Religion that acts in disguise and masquerade, changes frequently its colours, and puts out a false Flag to conceal the Pyrate.”

Similar uses of “false flag,” mostly in political writing, have continued into the 21st century. This is the most recent one in the OED: “These are the true Tory colours, not the false flag of convenience he flies for the working poor” (from the Daily Mirror, London, 2008).

As we said, it wasn’t until the 19th century that “false flag” showed up in the literal sense, defined in the OED as “a flag used to disguise a ship by misrepresenting its nationality, allegiance, intent, etc.”

Here is Oxford’s earliest example: “The boarding officers must, in their discretion, decide, whether this be a true or false flag, and of the character of the vessel” (from a May 29, 1824, article in the Daily National Intelligencer, Washington, DC, that refers to searching merchant vessels sailing under foreign flags).

And here’s the OED’s most recent literal citation: “The Obama administration is urging global port authorities to be on the watch for Iranian shipping vessels flying false flags or sailing under fraudulent registrations” (from an Associated Press article that appeared in print and online on July 19 and 20, 2012).

As Oxford notes, the term “false flag” is also used adjectivally, as in “false flag operation” (first recorded in 1982) and “false flag provocation” (2002).

In this sense, the dictionary says, the term describes “an event or action (typically political or military in nature) secretly orchestrated by someone other than the person or organization that appears to be responsible for it.”

Here’s the OED’s latest citation: “Some of those who believe that the 7/7 London bombings were a ‘false flag’ operation by state forces rejects [sic] as fake all the state’s evidence” (from a British monthly, Fortean Times, May 23, 2011).

More recently, an April 3, 2018, article in the Los Angeles Times about the Parkland, FL, high school shootings says “conspiracy theorists deemed the incident itself a hoax, or false flag, something that’s further marred the aftermath of every major shooting, including at Sandy Hook Elementary.”

As in those examples, the phrase is often used by people who argue that widely publicized mass shootings, bombings, and so on are actually “false flags” or “false flag operations,” staged by the government or interest groups for political purposes.

Help support the Grammarphobia Blog with your donation.
And check out our books about the English language.

Subscribe to the Blog by email

Enter your email address to subscribe to the Blog by email. If you are an old subscriber and not getting posts, please subscribe again.

Categories
English Etymology Expression Language Usage Word origin Writing

Can ‘refute’ mean ‘deny’?

Q: The increasing use of “refute” as a synonym for “deny” is threatening to finally separate me from the last vestiges of my sanity. I have always thought the former to mean “disprove,” which is very different than “deny.” I just read such an example in an Alaskan news article that sent me running to you.

A: You’ll be disappointed by our answer. Historically, “refute” and “deny” have had separate meanings. But over the last half-century or so the distinction has blurred, and today the use of “refute” to mean “deny” is accepted as standard English—at least by the editors of all nine dictionaries we’ve checked.

However, you’re not alone in objecting to the newer usage. Even some of the standard dictionaries that accept this sense acknowledge that there’s still resistance to it. And none of the usage manuals we’ve consulted wholeheartedly endorse the new sense, though some see the writing on the wall.

Here, for instance, are the senses of “refute” in The American Heritage Dictionary of the English Language (5th ed.):

(1) “To prove to be false or erroneous; overthrow by argument or proof,” as in “refute testimony.”

(2) “To deny the accuracy or truth of,” as in “refuted the results of the poll.”

Both are accepted as standard, but the dictionary adds this in a usage note:

“This second use has been criticized as incorrect or inappropriate since the early 1900s, despite being common. A majority of the Usage Panel accepts the use as a synonym of deny, but not by a wide margin. In our 2002 survey, 62 percent accepted the example In the press conference, the senator categorically refuted the charges of malfeasance but declined to go into details. This suggests that many readers are uncomfortable with this usage and would prefer to see deny in these contexts.”

Merriam-Webster Unabridged has similar definitions but adds that the second meaning “has frequently been cited as an error by commentators on English usage.”

The same is true with Webster’s New World College Dictionary (5th ed.), which accepts both meanings, but adds that the “deny” sense of “refute” is a “usage objected to by some.”

However, Merriam-Webster.com, an updated and expanded version of Merriam-Webster’s Collegiate Dictionary (11th ed.), accepts both meanings as standard, and without comment.

Similar definitions, without caveats, appear in four of the standard British dictionaries we consulted—Collins, LongmanMacmillan, and Cambridge online. All accept without reservation that “refute” can mean either to prove that something is untrue or to say that something is untrue.

Another British reference, Oxford Dictionaries online, accepts the new and old senses of “refute,” but adds a caveat similar to ones in American dictionaries.

Just because the newer sense is standard, though, doesn’t mean you have to use it. We don’t use “refute” to mean “deny.” The traditional sense is also standard, and that’s the one we use. However, you’ll have to endure the use of the newer sense by others.

For the time being, you’ll find support in most usage guides. Merriam-Webster’s Dictionary of English Usage, for example, notes that “refute” has “two senses, both of which are in common use, but one of which is widely regarded as an error.”

The usage guide notes, however, that the disputed use of “refute” to mean “deny” is “extremely common, and the contexts in which it occurs are standard”—that is, in speech or writing widely accepted as correct.

“Its most frequent use is by journalists reporting the emphatic denials issued by those accused of wrongdoing,” the usage guide adds. “Hardly a day now goes by, it seems, without one government official or another refuting a new set of allegations.”

In The Cambridge Guide to English Usage (2004), the linguist Pam Peters says “it seems unlikely that the objections can be sustained much longer in the face of usage. It may rankle with those who like to keep words in the state to which they are accustomed, but language moves on.”

We suspect that the traditional use of “refute” to mean “disprove” may be lost as the verb is increasingly used to mean “deny.” At some point, even traditionalists may have to abandon “refute” and use a term like “disprove” or “rebut” to make sure they’re understood.

The Oxford English Dictionary, an etymological dictionary, says English borrowed “refute” in the early 16th century from Anglo-Norman and Middle French, where refuter meant to prove something wrong, contest something, or reject someone. The ultimate source is refūtāre, classical Latin meaning to suppress or disprove something, or to prove someone wrong.

The verb meant to refuse or reject someone or something when it first appeared in English, according to the OED. The dictionary’s earliest citation is from the Benedictine monk Henry Bradshaw’s biography of St. Werburgh, patron saint of the author’s monastery in Chester, England:

“Her royall dyademe, and shynynge coronall / Was fyrst refuted, for loue of our sauyoure” (from The Holy Lyfe and History of Saynt Werburge, written sometime before Bradshaw’s death in 1513 and published posthumously in 1521). An Anglo-Saxon princess, Werburgh gave up her coronet to become an abbess.

Two decades after Bradshaw’s death, the verb took on what is now considered its traditional sense, which the OED defines as to “prove (something) to be false, esp. by means of argument or debate.”

The first Oxford example is from a 1533 polemic that Thomas More wrote during a theological debate with William Tyndale: “If Tyndale wold now refute myne obieccion of ye Turkes and theyr Alcharon.”

The use of “refute” to mean “deny” showed up in the 19th century. The earliest OED citation is from an 1886 satire of poor English: “Mind, i ain’t a snob; I utterly refute that idear. I don’t judge bi the koat he wares, or the joolery, or nothing of that kind.”

The dictionary’s next example is from a Canadian newspaper, the Manitoba Morning Free Press, Jan. 13, 1895: “Members wish to refute the assertions … that Hayes council ‘is on its last legs.’ Never in the history of the council was it in better shape.”

We’ve seen several possible earlier sightings, though it’s often hard to tell whether “refute” is being used in the sense of “disprove” or “deny.” As the OED notes, “In many instances it is unclear whether there is an implication of argument accompanying the assertion that something is baseless.”

In Jane Austen’s 1815 novel Emma, for example, Emma wonders why she doesn’t like Jane Fairfax:

“Mr. Knightley had once told her it was because she saw in her the really accomplished young woman, which she wanted to be thought herself; and though the accusation had been eagerly refuted at the time, there were moments of self-examination in which her conscience could not quite acquit her.”

And in The Warden (1855), the first of Anthony Trollope’s Barsetshire novels, Eleanor Harding is asked if she’s in love with John Bold:

“ ‘I—’ commenced Eleanor, turning sharply round to refute the charge; but the intended falsehood stuck in her throat, and never came to utterance. She could not deny her love, so she took plentifully to tears.” (The excerpt is from a conversation between Eleanor, the warden’s daughter, and Bold’s sister, Mary.)

The earliest objection to the newer usage, according to the M-W usage manual, is from Every-Day Words and Their Uses, a 1916 usage guide by Robert Palfrey Utter:

“To refute a statement, opinion, accusation, imputation, or charge, is not merely to call it in question, or deny it without proof, but to disprove it, overthrow it by argument, show it to be false.”

Henry Fowler doesn’t mention “refute” in A Dictionary of Modern English Usage (1926), which suggests that the new sense was uncommon at the time. But Sir Ernest Gowers, editor of the 1965 second edition of Fowler’s guide, says one refutes something “only by producing the evidence.” Without evidence, one “could only deny it.”

In the 1996 third edition, R. W. Burchfield writes, “I have an uneasy feeling that the new sense will begin to sound normal in the 21 c.—but not yet.”

And in the 2015 fourth edition, Jeremy Butterfield says that “it will sound normal to those who normally use it in this way, and aberrant to those who do not.” We assume it sounds aberrant to him. But the century is still young.

Help support the Grammarphobia Blog with your donation.
And check out our books about the English language.

Subscribe to the Blog by email

Enter your email address to subscribe to the Blog by email. If you are an old subscriber and not getting posts, please subscribe again.

Categories
English English language Etymology Expression Language Phrase origin Usage Word origin Writing

The life of ‘lifestyle’

Q: The more student papers I read at my university, the more certain usages drive me crazy. For decades I have resisted “lifestyle.” As Alfonse, a professor, says in Don DeLillo’s novel White Noise, “Californians invented the concept of life-style. This alone warrants their doom.” I believe he’s right on the latter point, but what about the former?

A: You may have to blame the Germans, not the Californians, for the all-too-common “lifestyle”—or at least for its earliest and rather isolated appearance in English, according to the Oxford English Dictionary.

However, we have our own opinions about the origin of the contemporary term, which we’ll discuss later.

The word is “perhaps” modeled after the German Lebensstil, the OED says. The German word, which dates from 1849, is a compound of leben (“life”) and stil (“style”), Oxford explains.

The earliest known example in English appeared in 1915, when a British philosopher, reviewing a book written in German, translated Lebensstil as “life-style.” Here’s the passage, as cited in the OED:

“This spirit of expediency … excludes any possibility of peace or rest in unity with the universe. The author applies to it, as the ‘life-style’ of our age, the term Impressionism.” (From the January 1915 issue of the journal Mind, where Bernard Bosanquet reviews Emil Hammacher’s Hauptfragen der Modernen Kultur, which means “main questions of modern culture.”)

The German book, published a year earlier, has Lebensstil twice, in the phrases “der Lebensstil unseres Zeitalters” (“the lifestyle of our age”) and “den impressionistischen Lebensstil” (“the Impressionist lifestyle”).

The OED defines this use of “lifestyle” as “a style or way of living (associated with an individual person, a society, etc.); esp. the characteristic manner in which a person lives (or chooses to live) his or her life.”

It’s defined more briefly in Merriam Webster Unabridged—”the typical way of life of an individual, group, or culture”—and illustrated with the phrases “a healthy lifestyle” and “an alternative lifestyle.”

We haven’t found any earlier examples of the term, whether written as one word (“lifestyle”), two (“life style”), or hyphenated (“life-style”). And even after 1915, it didn’t catch on for several decades.

The term in this sense—a way of living—wasn’t sighted again until 1947 (the OED also has a 1946 example, but it’s for a different meaning, as we’ll explain later). Here’s the 1947 citation:

“While ostensibly setting about the freeing of the slaves, they became enslaved, and found in the wailing self-pity and crooning of the Negro the substitute for any life-style of their own” (from an article by Marshall McLuhan in the October 1947 issue of the Sewanee Review).

The term wasn’t recorded again, as far as we can tell, until the 1960s, when it became widely known.

The next OED citation is from the March 22, 1961, issue of the Guardian, London: “The mass-media … continually tell their audience what life-styles are ‘modern’ and ‘smart.’ ”

Today the word has become so common that many consider it trite. The New York Times Manual of Style and Usage (5th ed.) regards it as “shopworn.”

And, more to the point of your question, “lifestyle” has become associated with consumerism and conspicuous consumption. We might ponder the “lifestyle” of the Kardashians, but it would be jarring to write a college paper about the “lifestyle” of prisoners in the gulag.

Since the 1970s, the OED says, “lifestyle” has also been used as an adjective, describing something “designed to appeal to consumers by association with a lifestyle regarded as desirable, glamorous, or attractive.” Oxford uses examples like “lifestyle advertising,” “lifestyle brand,” and “lifestyle magazine.”

But we have some more thoughts on the origins of the noun that means a way of living.

Certainly the earliest “lifestyle” on record, that 1915 example, was modeled after the German Lebensstil; it was a direct translation.

However, we wonder whether the later use of the word in ordinary English wasn’t simply spontaneous. We say this for two reasons:

(1) That early appearance in a review of an obscure German book seems unlikely to have inspired either the isolated 1947 example or the surge in the use of “lifestyle” beginning in the 1960s.

(2) The concept itself—a way of living—dates from an earlier time, when it was commonly expressed as “style of living” or “style of life.” And it seems likely that those phrases inspired the shorter, catchier “lifestyle.”

We’ve found many examples of “style of living” from the late 18th century onward, and of “style of life” from the mid-19th-century. These phrases—meaning the same thing as the modern “lifestyle”—were and still are found on both sides of the Atlantic, though they’re not nearly as common as they once were.

For instance, here are some early uses of the older of the two phrases, “style of living”:

1784: “You can conceive nothing so charming as the Grecian women!—nothing so interesting as their style of living!” (From More Ways Than One, a comic play by Hannah Cowley.)

1785: “In a few years she resumed her equipage, and recommenced her usual style of living, with as much or rather more splendor than ever.” (A Philosophical, Historical, and Moral Essay on Old Maids, by William Hayley.)

1785: “This [Switzerland] is not a cheap country. … In some respects my style of living is enlarged by the increase of my relative importance, an obscure bachelor in England, the master of a considerable house at Lausanne.” (From a letter written on March 21 by Edward Gibbon to Lord Sheffield.)

1793: “His style of living is not equal to his fortune; and I have heard of several instances of his attention to petty economy.” (Evenings at Home, Vol. III, a collection of children’s pieces by Anna Laetitia Barbauld and John Aikin.)

1794: “ ‘Our notions are somewhat enlarged since those days,’ said M. Quesnel;—‘what was then thought a decent style of living would not now be endured.’ ” (The Mysteries of Udolpho, a novel by Ann Radcliffe.)

1797: “In food little luxury seems to have been known, till James I, who had resided nineteen years in England, set the example of a higher style of living.” (A History of Scotland, by John Pinkerton.)

By the mid-19th century, “style of living” was so familiar a phrase in British and American literature that some writers were overusing it (as they would later overuse “lifestyle”).

For instance, in Six Lectures Addressed to the Working Classes (1849), the Scottish minister William G. Blaikie writes: “It is very desirable that the working classes stood higher in the esteem of the community, and enjoyed a more comfortable style of living.”  Later he goes on to use phrase six times on a single page—”a very wretched style of living” … “a thirst for a higher style of living” … “the Irish style of living” … “the desire of improving his style of living,” and so on.

If those authors were writing today, our bet is that they’d use “lifestyle” instead.

A search of Google Ngram Viewer, which tracks words and phrases in millions of books, shows that “lifestyle” sprang to life in the mid-1960s and spiked sharply, just when “style of living” and “style of life” dropped away. You might almost say that “lifestyle” replaced them

So we suspect that the “lifestyle” that vaulted into use in the mid-20th century was a product of those earlier phrases, not of Lebensstil.

Before we go, we should mention a very different meaning of “lifestyle” that’s much less common and not known to the average reader.

In the psychology of Alfred Adler, the OED explains, it means “a pattern of reactions and behaviour that is established in childhood and remains characteristic of an individual.”

This technical meaning of “lifestyle” came into English in 1929 from Lebensstil, the OED says. The German word used as a psychological term, Oxford says, was first recorded in “1928 or earlier.”

The OED‘s earliest English example is from Adler’s Problems of Neurosis, which he wrote in English and published in 1929: “This fragment of memory records the two typical motives of the main life-style.”

The 1946 example we mentioned above, which the OED cites for the other sense of “lifestyle,” belongs in this category, in our opinion. The author, George Orwell, used the word in the Adlerian sense to mean a pattern of behavior:

“True to his life-style, Koestler was … promptly arrested and interned by the Daladier Government.” (From Critical Essays.)

Help support the Grammarphobia Blog with your donation.
And check out our books about the English language.

Subscribe to the Blog by email

Enter your email address to subscribe to the Blog by email. If you are an old subscriber and not getting posts, please subscribe again.

Categories
English English language Expression Grammar Language Linguistics Usage Writing

When ‘I have’ becomes ‘I’ve’

Q: I’m seeing the use of “I’ve” where “have” is not an auxiliary but the actual verb, as in “I’ve a red car.” That’s not legit, is it? Yuck.

A: We’ve searched the major scholarly works of grammar and haven’t found any grammatical objections to using “I’ve” as a contraction when “have” is the primary verb. However, the usage is more common in Britain and may sound unidiomatic to American ears.

In British or American English? A Handbook of Word and Grammar Patterns (2006), the linguist John Algeo writes that the use of “I’ve” with the contracted primary verb is over five times more frequent in the UK than in the US.

What you’re noticing, however, may be a shortening of the word “have” in speech or written dialogue, rather than an actual contraction. The “h” or “ha” sounds are often dropped in conversation, and the contraction itself is elided when an expression like “I’ve got to go” comes out as “I gotta go.”

As for the contraction, Sydney Greenbaum, writing in the Oxford English Grammar, describes “I’ve” as the contracted form of “I have” when “have” is either a primary verb or an auxiliary.

We couldn’t find any objections to the usage in The Cambridge Grammar of the English Language, by Rodney Huddleston and Geoffrey K. Pullum, or A Comprehensive Grammar of the English Language, by Randolph Quirk et al.

However, both books question the usage in some negative constructions.

Quirk, for example, says it’s generally preferable to contract “not” rather than “have” in such constructions, recommending “I haven’t” instead of “I’ve not.” Huddleston and Pullum consider sentences like “I’ve no time today” and “I haven’t enough tea” to be grammatical only in certain dialects.

We checked a half-dozen standard dictionaries and all of them define “I’ve” as simply a contraction for “I have.” None of them say it can’t be used to contract the principal verb. In fact, two of the dictionaries include examples that do just that:

“I’ve one more appointment today” (from Merriam-Webster Unabridged) and “I’ve no other appointments” (from the online Collins Dictionary).

Finally, none of the usage manuals in our extensive language library comment on the use of “I’ve” as a contraction for “I have,” suggesting that the authors aren’t especially bothered by the way “I’ve” is being used today in either in the US or the UK.

Help support the Grammarphobia Blog with your donation.
And check out our books about the English language.

Subscribe to the Blog by email

Enter your email address to subscribe to the Blog by email. If you are an old subscriber and not getting posts, please subscribe again.